Upper GI and Lower GI

¡Supera tus tareas y exámenes ahora con Quizwiz!

When evaluating the patient's understanding about the care of the ileostomy, which statement by the patient indicates the patient needs more teaching? "I will be able to regulate when I have stools." "I will be able to wear a pouch until it leaks." "The drainage from my stoma can damage my skin." "Dried fruit and popcorn must be chewed very well."

"I will be able to regulate when I have stools." An ileostomy is in the ileum and drains liquid stool frequently, unlike a colostomy, which has more formed stool the farther distal the ostomy is in the colon. The ileostomy pouch is usually worn for 4 to 7 days or until it leaks. It must be changed immediately if it leaks because the drainage is very irritating to the skin. To avoid obstruction, popcorn, dried fruit, coconut, mushrooms, olives, stringy vegetables, food with skin, and meats with casings must be chewed extremely well before swallowing because of the narrow diameter of the ileostomy lumen.

The nurse is preparing to administer a dose of bisacodyl to a patient with constipation and the patient asks how it will work. What is the best response by the nurse? "It will increase bulk in the stool." "It will lubricate the intestinal tract to soften feces." "It will increase fluid retention in the intestinal tract." "It will increase peristalsis by stimulating nerves in the colon wall."

"It will increase peristalsis by stimulating nerves in the colon wall." Bisacodyl is a stimulant laxative that aids in producing a bowel movement by irritating the colon wall and stimulating enteric nerves. Fiber and bulk-forming drugs increase bulk in the stool. Water and stool softeners soften feces, and saline and osmotic solutions cause fluid retention in the intestinal tract.

The nurse is preparing to administer famotidine to a patient after a laparotomy. The patient states they do not have heartburn. What response by the nurse would be the most appropriate? "It will prevent air from accumulating in the stomach, causing gas pains." "It will reduce the amount of acid in the stomach while you are not eating." "It will prevent the heartburn that occurs as a side effect of general anesthesia." "The stress of surgery is likely to cause stomach bleeding if you do not receive it."

"It will reduce the amount of acid in the stomach while you are not eating." Famotidine is an H2-receptor antagonist that inhibits gastric HCl secretion and thus minimizes damage to gastric mucosa while the patient is not eating a regular diet after surgery. Famotidine does not prevent air from accumulating in the stomach or stop the stomach from bleeding. Heartburn is not a side effect of general anesthesia.

The nurse is preparing to insert a nasogastric (NG) tube into a patient with a suspected small intestinal obstruction that is vomiting. The patient asks the nurse why this procedure is necessary. What response by the nurse is most appropriate? "The tube will help to drain the stomach contents and prevent further vomiting." "The tube will push past the area that is blocked and help to stop the vomiting." "The tube is just a standard procedure before many types of surgery to the abdomen." "The tube will let us measure your stomach contents so we can give you the right IV fluid replacement."

"The tube will help to drain the stomach contents and prevent further vomiting." Rationale: The NG tube is used to decompress the stomach by draining stomach contents and thereby prevent further vomiting. The NG tube will not push past the blocked area. Potential surgery is not currently indicated. The location of the obstruction will determine the type of fluid to use, not measure the amount of stomach contents.

A patient is given a bisacodyl suppository and asks the nurse how long it will take to work. What is the best response by the nurse? 2 to 5 minutes 15 to 60 minutes 2 to 4 hours 6 to 8 hours

15 to 60 minutes Bisacodyl suppositories usually are effective within 15 to 60 minutes of administration, so the nurse should plan accordingly to assist the patient to use the bedpan or commode.

A patient with suspected bowel obstruction had a nasogastric tube inserted at 4:00 AM. The nurse shares in the morning report that the day shift staff should check the tube for patency at what times? 7:00 AM, 10:00 AM, and 1:00 PM 8:00 AM, 12:00 PM, and 4:00 PM 9:00 AM and 3:00 PM 9:00 AM, 12:00 PM, and 3:00 PM

8:00 AM, 12:00 PM, and 4:00 PM : A nasogastric tube should be checked for patency routinely at 4-hour intervals. Thus if the tube were inserted at 4:00 AM, it would be due to be checked at 8:00 AM, 12:00 PM, and 4:00 PM.

The nurse identifies that which patient is at highest risk for developing colon cancer? A 28-yr-old man who has a body mass index of 27 kg/m2 A 32-yr-old woman with a 12-year history of ulcerative colitis A 52-yr-old man who has followed a vegetarian diet for 24 years A 58-yr-old woman taking prescribed estrogen replacement therapy

A 32-yr-old woman with a 12-year history of ulcerative colitis Risk for colon cancer includes personal history of inflammatory bowel disease (especially ulcerative colitis for longer than 10 years); obesity; family (first-degree relative) or personal history of colorectal cancer, adenomatous polyposis, or hereditary nonpolyposis colorectal cancer syndrome; eating red meat; cigarette use; and drinking alcohol.

A patient is planned for discharge home today after ostomy surgery for colon cancer. The nurse should assign the patient to which staff member? An UAP on the unit who has hospice experience An LPN that has worked on the unit for 10 years An RN with 6 months of experience on the surgical unit An RN who has floated to the surgical unit from pediatrics

An RN with 6 months of experience on the surgical unit The patient needs ostomy care directions and reinforcement at discharge and should be assigned to a registered nurse with experience in providing discharge teaching for ostomy care. Teaching should not be delegated to a LPN/VN or UAP.

Which clinical manifestations of inflammatory bowel disease are common to both patients with ulcerative colitis (UC) and Crohn's disease? (Select all that apply) Restricted to rectum Strictures are common Bloody, diarrhea stools Cramping abdominal pain Lesions penetrate intestine

Bloody, diarrhea stools Cramping abdominal pain Manifestations of UC and Crohn's disease include bloody diarrhea, cramping abdominal pain, and nutritional disorders. Intestinal lesions associated with UC are usually restricted to the rectum before moving into the colon. Lesions that penetrate the intestine or cause strictures are characteristic of Crohn's disease.

The nurse is preparing to administer a scheduled dose of docusate sodium when the patient reports an episode of loose stool and does not want to take the medication. What is the appropriate action by the nurse? Write an incident report about this untoward event. Attempt to have the family convince the patient to take the ordered dose. Withhold the medication at this time and try to administer it later in the day. Chart the dose as not given on the medical record and explain in the nursing progress notes.

Chart the dose as not given on the medical record and explain in the nursing progress notes. Whenever a patient refuses medication, the dose should be charted as not given with an explanation of the reason documented in the nursing progress notes. In this instance, the refusal indicates good judgment by the patient, and the patient should not be encouraged to take it today.

A patient is admitted to the emergency department after a motor vehicle crash with suspected abdominal trauma. What assessment finding by the nurse is of highest priority? Nausea and vomiting Hyperactive bowel sounds Firmly distended abdomen Abrasions on all extremities

Firmly distended abdomen Manifestations of abdominal trauma are guarding and splinting of the abdominal wall; a hard, distended abdomen (indicating possible intraabdominal bleeding); decreased or absent bowel sounds; contusions, abrasions, or bruising over the abdomen; abdominal pain; pain over the scapula; hematemesis or hematuria; and signs of hypovolemic shock (tachycardia and decreased blood pressure).

The nurse is caring for a patient who develops watery diarrhea and a fever after prolonged omeprazole (Prilosec) therapy. In which order will the nurse take actions? (Put a comma and a space between each answer choice [A, B, C, D].) a. Contact the health care provider. b. Assess blood pressure and heart rate. c. Give the PRN acetaminophen (Tylenol). d. Place the patient on contact precautions.

D, B, A, C Proton pump inhibitors including omeprazole (Prilosec) may increase the risk of Clostridium difficile-associated colitis. Because the patient's history and symptoms are consistent with C. difficile infection, the initial action should be initiation of infection control measures to protect other patients. Assessment of blood pressure and pulse is needed to determine whether the patient has symptoms of hypovolemia or shock. The health care provider should be notified so that actions such as obtaining stool specimens and antibiotic therapy can be started. Tylenol may be administered but is the lowest priority of the actions.

The nurse asks a patient scheduled for colectomy to sign the operative permit as directed in the provider's preoperative orders. The patient states that the provider has not explained very well what is involved in the surgical procedure. What is the most appropriate action by the nurse? a. Ask family members whether they have discussed the surgical procedure with the provider. b. Explain the planned surgical procedure as well as possible and have the patient sign the consent form. c. Have the patient sign the form and state the provider will visit to explain the procedure before surgery. d. Delay the patient's signature on the consent and notify the provider about the conversation with the patient.

Delay the patient's signature on the consent and notify the provider about the conversation with the patient. The patient should not be asked to sign a consent form unless the procedure has been explained to the satisfaction of the patient. The nurse should notify the provider, who has the responsibility for obtaining consent.

The nurse is admitting a patient with severe dehydration and frequent watery diarrhea. A 10-day outpatient course of antibiotic therapy for bacterial pneumonia has just been completed. What is the most important for the nurse to take which action? Wear a mask to prevent transmission of infection. Have visitors use the alcohol-based hand sanitizer. Wipe down equipment with ammonia-based disinfectant. Don gloves and gown before entering the patient's room.

Don gloves and gown before entering the patient's room. Clostridium difficile is an antibiotic-associated diarrhea transmitted by contact, and the spores are extremely difficult to kill. Patients with suspected or confirmed infection with C. difficile should be placed in a private room, and gloves and gowns should be worn by visitors and health care providers. Alcohol-based hand cleaners and ammonia-based disinfectants are ineffective and do not kill all the spores. Equipment cannot be shared with other patients, and a disposable stethoscope and individual patient thermometer are kept in the room. Objects should be disinfected with a 10% solution of household bleach.

When teaching the patient about the diet for diverticular disease, which foods should the nurse recommend? White bread, cheese, and green beans Fresh tomatoes, pears, and corn flakes Oranges, baked potatoes, and raw carrots Dried beans, All Bran (100%) cereal, and raspberries

Dried beans, All Bran (100%) cereal, and raspberries A high-fiber diet is recommended for diverticular disease. Dried beans, All Bran (100%) cereal, and raspberries all have higher amounts of fiber than white bread, cheese, green beans, fresh tomatoes, pears, corn flakes, oranges, baked potatoes, and raw carrots.

After an exploratory laparotomy, a patient on a clear liquid diet reports severe gas pains and abdominal distention. Which action by the nurse is most appropriate? Return the patient to NPO status. Place cool compresses on the abdomen. Encourage the patient to ambulate as ordered. Administer an as-needed dose of IV morphine sulfate.

Encourage the patient to ambulate as ordered. Swallowed air and reduced peristalsis after surgery can result in abdominal distention and gas pains. Early ambulation helps restore peristalsis and eliminate flatus and gas pain. Medications used to reduce gas pain include metoclopramide, which stimulates peristalsis. A heating pad can help to alleviate some of the pain and help make the patient more comfortable. There is no need for the patient to return to NPO status. Drinking ginger ale may be helpful.

A patient after a stroke who primarily uses a wheelchair for mobility has developed diarrhea with fecal incontinence. What is a priority assessment by the nurse? Fecal impaction Perineal hygiene Dietary fiber intake Antidiarrheal agent use

Fecal impaction Patients with limited mobility are at risk for fecal impactions caused by constipation that may lead to liquid stool leaking around the hardened impacted feces, so assessing for fecal impaction is the priority. Perineal hygiene can be assessed at the same time. Assessing the dietary fiber and fluid intake and antidiarrheal agent use will be assessed and considered next

The nurse is caring for a patient admitted with a suspected bowel obstruction. The nurse auscultating the abdomen listens for which type of bowel sounds that are consistent with the patient's clinical picture? Low-pitched and rumbling above the area of obstruction High-pitched and hypoactive below the area of obstruction Low-pitched and hyperactive below the area of obstruction High-pitched and hyperactive above the area of obstruction

High-pitched and hyperactive above the area of obstruction Early in intestinal obstruction, the patient's bowel sounds are hyperactive and high pitched, sometimes referred to as "tinkling," above the level of the obstruction. This occurs because peristaltic action increases to "push past" the area of obstruction. As the obstruction becomes complete, bowel sounds decrease and finally become absent.

The nurse is developing a plan of care for a patient with an abdominal mass and suspected bowel obstruction. Which factor in the patient's history does the nurse recognize as increasing the patient's risk for colorectal cancer? Osteoarthritis History of colorectal polyps History of lactose intolerance Use of herbs as dietary supplements

History of colorectal polyps Rationale: A history of colorectal polyps places this patient at risk for colorectal cancer. This tissue can degenerate over time and become malignant. Osteoarthritis, lactose intolerance, and the use of herbs do not pose additional risk to the patient.

What information would have the highest priority for the nurse to include in preoperative teaching for a patient scheduled for an exploratory laparotomy? How to care for the wound How to deep breathe and cough The location and care of drains after surgery Which medications will be used during surgery

How to deep breathe and cough Because anesthesia, an abdominal incision, and pain can impair the patient's respiratory status in the postoperative period, it is of high priority to teach the patient to cough and deep breathe. Otherwise, the patient could develop atelectasis and pneumonia, which would delay early recovery from surgery and hospital discharge. Care for the wound and location and care of the drains will be briefly discussed preoperatively but will be done again with higher priority after surgery. Knowing which drugs will be used during surgery may not be meaningful to the patient and should be reviewed with the patient by the anesthesiologist.

Two days after a bowel resection for an abdominal mass, a patient reports gas pains and abdominal distention. The nurse plans care for the patient based on the knowledge that the symptoms are occurring as a result what event? Impaired peristalsis Irritation of the bowel Nasogastric suctioning Inflammation of the incision site

Impaired peristalsis Until peristalsis returns to normal after anesthesia, the patient may experience slowed gastrointestinal motility, leading to gas pains and abdominal distention. Irritation of the bowel, nasogastric suctioning, and inflammation of the surgical site do not cause gas pains or abdominal distention.

A hospitalized patient has just been diagnosed with diarrhea due to C. difficile. Which nursing interventions should be included in the patient's plan of care? (Select all that apply.) Initiate contact isolation precautions. Place the patient on a clear liquid diet. Teach any visitors to wear gloves and gowns. Disinfect the room with 10% bleach solution as needed. Use hand sanitizer before and after any bodily fluid contact.

Initiate contact isolation precautions. Teach any visitors to wear gloves and gowns. Disinfect the room with 10% bleach solution as needed. Initiation of contact isolation precautions must be done immediately with a patient with C. difficile, which includes washing hands with soap and water before and after patient or bodily fluid contact. Alcohol-based sanitizers are ineffective. Visitors need to be taught to wear gloves and gowns and wash hands. A clear liquid diet is not necessary. The room will be disinfected with 10% bleach solution when the patient is dismissed and may be done periodically during the patient's stay, depending on the agency policy.

The nurse is administering a cathartic agent to a patient with renal insufficiency. Which order will the nurse question? Bisacodyl Lubiprostone Cascara sagrada Magnesium hydroxide

Magnesium hydroxide Milk of magnesia may cause hypermagnesemia in patients with renal insufficiency. The nurse should question this order with the health care provider. Bisacodyl, lubiprostone, and cascara sagrada are safe to use in patients with renal insufficiency as long as the patient is not currently dehydrated.

The nurse is conducting discharge teaching for a patient with metastatic lung cancer who was admitted with a bowel impaction. Which instructions would be most helpful to prevent further episodes of constipation? Maintain a high intake of fluid and fiber in the diet. Discontinue intake of medications causing constipation. Eat several small meals per day to maintain bowel motility. Sit upright during meals to increase bowel motility by gravity.

Maintain a high intake of fluid and fiber in the diet. Increased fluid intake and a high-fiber diet reduce the incidence of constipation caused by immobility, medications, and other factors. Fluid and fiber provide bulk that in turn increases peristalsis and bowel motility. Analgesics taken for lung cancer probably cannot be discontinued. Eating several small meals per day and position do not facilitate bowel motility.

The nurse should administer an as-needed dose of magnesium citrate after noting what information when reviewing a patient's medical record? Abdominal pain and bloating No bowel movement for 3 days A decrease in appetite by 50% over 24 hours Muscle tremors and other signs of hypomagnesemia

No bowel movement for 3 days Magnesium citrate is an osmotic laxative that produces a soft, semisolid stool usually within 15 minutes to 3 hours. This medication would benefit the patient who has not had a bowel movement for 3 days. It would not be given for abdominal pain and bloating, decreased appetite, or signs of hypomagnesemia.

A patient with an intestinal obstruction has a nasogastric (NG) tube to suction but reports of nausea and abdominal distention. The nurse irrigates the tube as necessary as ordered, but the irrigating fluid does not return. What should be the priority action by the nurse?Notify the provider. Auscultate for bowel sounds. Reposition the tube and check for placement. Remove the tube and replace it with a new one.

Reposition the tube and check for placement. The tube may be resting against the stomach wall. The first action by the nurse is to reposition the tube and check it again for placement. The provider does not need to be notified unless the nurse cannot restore the tube function. The patient does not have bowel sounds, which is why the NG tube is in place. The NG tube would not be removed and replaced unless it was no longer in the stomach or the obstruction of the tube could not be relieved.

A patient with ulcerative colitis is scheduled for a colon resection with placement of an ostomy. The nurse should plan to include which prescribed measure in the preoperative preparation? Selecting the stoma site Where to purchase ostomy supplies Teaching about how to irrigate a colostomy Following a high-fiber diet the day before surgery

Selecting the stoma site Care that is unique to ostomy surgery includes selecting the best site for the stoma. Instructions to irrigate the colostomy and where to purchase ostomy supplies will be done postoperatively. A clear liquid diet will be used the day before surgery with the bowel cleansing.

The nurse is preparing to administer a daily dose of docusate sodium to a patient that will continue taking it after discharge. What information should the nurse provide to the patient to optimize the outcome of the medication? Take a dose of mineral oil at the same time. Add extra salt to food on at least one meal tray. Ensure a dietary intake of 10 g of fiber each day. Take each dose with a full glass of water or other liquid.

Take each dose with a full glass of water or other liquid. Docusate lowers the surface tension of stool, permitting water and fats to penetrate and soften the stool for easier passage. The patient should take the dose with a full glass of water and should increase overall fluid intake, if able, to enhance effectiveness of the medication. Dietary fiber intake should be a minimum of 20 g daily to prevent constipation. Mineral oil and extra salt are not recommended.

A patient with ulcerative colitis is scheduled for a total proctocolectomy with permanent ileostomy. The wound, ostomy, and continence nurse is selecting the site where the ostomy will be placed. What should be included in site consideration? Protruding areas make the best sites. The patient must be able to see the site. The site should be outside the rectus muscle area. The appliance will need to be placed at the waist line.

The patient must be able to see the site. In selection of the ostomy site, the WOC nurse will want a site visible to the patient so the patient can take care of it, within the rectus muscle to avoid hernias, and on a flat surface to more easily create a good seal with the drainage bag. Care should be taken to avoid skin creases, scars, and belt lines, which can interfere with the adherence of the appliance

A patient with a new ileostomy asks how much drainage to expect. The nurse explains that after the bowel adjusts to the ileostomy, the usual drainage will be about _____ cups daily. a. 2 b. 3 c. 4 d. 5

a. 2 After the proximal small bowel adapts to reabsorb more fluid, the average amount of ileostomy drainage is about 500 mL daily. One cup is about 240 mL.

The nurse is assessing a patient with abdominal pain. The nurse, who notes that there is ecchymosis around the area of umbilicus, will document this finding as a. Cullen sign. b. Rovsing sign. c. McBurney sign. d. Grey-Turner's sign.

a. Cullen sign. Cullen sign is ecchymosis around the umbilicus. Rovsing sign occurs when palpation of the left lower quadrant causes pain in the right lower quadrant. Grey Turner's sign is bruising over the flanks. Deep tenderness at McBurney's point (halfway between the umbilicus and the right iliac crest), known as McBurney's sign, is a sign of acute appendicitis.

A patient complains of gas pains and abdominal distention 2 days after a small bowel resection. Which nursing action should the nurse take? a. Encourage the patient to ambulate. b. Instill a mineral oil retention enema. c. Administer the prescribed IV morphine sulfate. d. Offer the prescribed promethazine (Phenergan).

a. Encourage the patient to ambulate. Ambulation will improve peristalsis and help the patient eliminate flatus and reduce gas pain. A mineral oil retention enema is helpful for constipation with hard stool. A return-flow enema might be used to relieve persistent gas pains. Morphine will further reduce peristalsis. Promethazine is used as an antiemetic rather than to decrease gas pains or distention.

Which nursing action will be included in the plan of care for a 25-yr-old male patient with a new diagnosis of irritable bowel syndrome (IBS)? a. Encourage the patient to express concerns and ask questions about IBS. b. Suggest that the patient increase the intake of milk and other dairy products. c. Teach the patient to avoid using nonsteroidal antiinflammatory drugs (NSAIDs). d. Teach the patient about the use of alosetron (Lotronex) to reduce IBS symptoms.

a. Encourage the patient to express concerns and ask questions about IBS. Because psychologic and emotional factors can affect the symptoms for IBS, encouraging the patient to discuss emotions and ask questions is an important intervention. Alosetron has serious side effects and is used only for female patients who have not responded to other therapies. Although yogurt may be beneficial, milk is avoided because lactose intolerance can contribute to symptoms in some patients. NSAIDs can be used by patients with IBS.

A patient with Crohn's disease who is taking infliximab (Remicade) calls the nurse in the outpatient clinic about new symptoms. Which symptom is most important to communicate to the health care provider? a. Fever b. Nausea c. Joint pain d. Headache

a. Fever Since infliximab suppresses the immune response, rapid treatment of infection is essential. The other patient complaints are common side effects of the medication, but they do not indicate any potentially life-threatening complications.

Which menu choice by the patient with diverticulosis is best for preventing diverticulitis? a. Navy bean soup and vegetable salad b. Whole grain pasta with tomato sauce c. Baked potato with low-fat sour cream d. Roast beef sandwich on whole wheat bread

a. Navy bean soup and vegetable salad A diet high in fiber and low in fats and red meat is recommended to prevent diverticulitis. Although all of the choices have some fiber, the bean soup and salad will be the highest in fiber and the lowest in fat.

A patient is awaiting surgery for acute peritonitis. Which action will the nurse include in the plan of care? a. Position patient with the knees flexed. b. Avoid use of opioids or sedative drugs. c. Offer frequent small sips of clear liquids. d. Assist patient to breathe deeply and cough.

a. Position patient with the knees flexed. There is less peritoneal irritation with the knees flexed, which will help decrease pain. Opioids and sedatives are typically given to control pain and anxiety. Preoperative patients with peritonitis are given IV fluids for hydration. Deep breathing and coughing will increase the patient's discomfort.

Which prescribed intervention for a patient with chronic short bowel syndrome will the nurse question? a. Senna 1 tablet every day b. Ferrous sulfate 325 mg daily c.Psyllium (Metamucil) 3 times daily d. Diphenoxylate with atropine (Lomotil) prn loose stools

a. Senna 1 tablet every day Patients with short bowel syndrome have diarrhea because of decreased nutrient and fluid absorption and would not need stimulant laxatives. Iron supplements are used to prevent iron-deficiency anemia, bulk-forming laxatives help make stools less watery, and opioid antidiarrheal drugs are helpful in slowing intestinal transit time.

The nurse admitting a patient with acute diverticulitis explains that the initial plan of care is to a. administer IV fluids. b. prepare for colonoscopy. c. give stool softeners and enemas. d. order a diet high in fiber and fluids.

a. administer IV fluids. A patient with acute diverticulitis will be NPO and given parenteral fluids. A diet high in fiber and fluids will be implemented before discharge. Bulk-forming laxatives, rather than stool softeners, are usually given, and these will be implemented later in the hospitalization. The patient with acute diverticulitis will not have enemas or a colonoscopy because of the risk for perforation and peritonitis.

A patient calls the clinic to report a new onset of severe diarrhea. The nurse anticipates that the patient will need to a. collect a stool specimen. b. prepare for colonoscopy. c. schedule a barium enema. d. have blood cultures drawn.

a. collect a stool specimen. Acute diarrhea is usually caused by an infectious process, and stool specimens are obtained for culture and examined for parasites or white blood cells. There is no indication that the patient needs a colonoscopy, blood cultures, or a barium enema.

A patient has a new diagnosis of Crohn's disease after having frequent diarrhea and a weight loss of 10 lb (4.5 kg) over 2 months. The nurse will plan to teach about a. medication use. b. fluid restriction. c. enteral nutrition. d. activity restrictions.

a. medication use. Medications are used to induce and maintain remission in patients with inflammatory bowel disease (IBD). Decreased activity level is indicated only if the patient has severe fatigue and weakness. Fluids are needed to prevent dehydration. There is no advantage to enteral feedings.

The nurse is admitting a 67-yr-old patient with new-onset steatorrhea. Which question is most important for the nurse to ask? a. "How much milk do you usually drink?" b. "Have you noticed a recent weight loss?" c. "What time of day do your bowels move?" d. "Do you eat meat or other animal products?"

b. "Have you noticed a recent weight loss?" Although all of the questions provide useful information, it is most important to determine if the patient has an imbalance in nutrition because of the steatorrhea.

Which patient statement indicates that the nurse's teaching about sulfasalazine (Azulfidine) for ulcerative colitis has been effective? a. "The medication will be tapered if I need surgery." b. "I will need to use a sunscreen when I am outdoors." c. "I will need to avoid contact with people who are sick." d. "The medication prevents the infections that cause diarrhea."

b. "I will need to use a sunscreen when I am outdoors." Sulfasalazine may cause photosensitivity in some patients. It is not used to treat infections. Sulfasalazine does not reduce immune function. Unlike corticosteroids, tapering of sulfasalazine is not needed.

After change-of-shift report, which patient should the nurse assess first? a. A 40-yr-old male patient with celiac disease who has frequent frothy diarrhea b. A 30-yr-old female patient with a femoral hernia who has abdominal pain and vomiting c. A 30-yr-old male patient with ulcerative colitis who has severe perianal skin breakdown d. A 40-yr-old female patient with a colostomy bag that is pulling away from the adhesive wafer

b. A 30-yr-old female patient with a femoral hernia who has abdominal pain and vomiting Pain and vomiting with a femoral hernia suggest possible strangulation, which will necessitate emergency surgery. The other patients have less urgent problems

After a total proctocolectomy and permanent ileostomy, the patient tells the nurse, "I cannot manage all these changes. I don't want to look at the stoma." What is the best action by the nurse? a. Reassure the patient that ileostomy care will become easier. b. Ask the patient about the concerns with stoma management. c. Postpone any teaching until the patient adjusts to the ileostomy. d. Develop a detailed written list of ostomy care tasks for the patient.

b. Ask the patient about the concerns with stoma management. Encouraging the patient to share concerns assists in helping the patient adjust to the body changes. Acknowledgment of the patient's feelings and concerns is important rather than offering false reassurance. Because the patient indicates that the feelings about the ostomy are the reason for the difficulty with the many changes, development of a detailed ostomy care plan will not improve the patient's ability to manage the ostomy. Although detailed ostomy teaching may be postponed, the nurse should offer teaching about some aspects of living with an ostomy.

A 25-yr-old male patient calls the clinic complaining of diarrhea for 24 hours. Which action should the nurse take first? a. Inform the patient that laboratory testing of blood and stools will be necessary. b. Ask the patient to describe the character of the stools and any associated symptoms. c. Suggest that the patient drink clear liquid fluids with electrolytes, such as Gatorade or Pedialyte. d. Advise the patient to use over-the-counter loperamide (Imodium) to slow gastrointestinal (GI) motility.

b. Ask the patient to describe the character of the stools and any associated The initial response by the nurse should be further assessment of the patient. The other responses may be appropriate, depending on what is learned in the assessment.

Which information will the nurse include when teaching a patient how to avoid chronic constipation (select all that apply)? a. Stimulant and saline laxatives can be used regularly. b. Bulk-forming laxatives are an excellent source of fiber. c. Walking or cycling frequently will help bowel motility. d. A good time for a bowel movement may be after breakfast. e. Some over-the-counter (OTC) medications cause constipation.

b. Bulk-forming laxatives are an excellent source of fiber. c. Walking or cycling frequently will help bowel motility. d. A good time for a bowel movement may be after breakfast. e. Some over-the-counter (OTC) medications cause constipation. Stimulant and saline laxatives should be used infrequently. Use of bulk-forming laxatives, regular early morning timing of defecation, regular exercise, and avoiding many OTC medications will help the patient avoid constipation.

A young woman who has Crohn's disease develops a fever and symptoms of a urinary tract infection (UTI) with tan, fecal-smelling urine. What information will the nurse add to a general teaching plan about UTIs in order to individualize the teaching for this patient? a. Bacteria in the perianal area can enter the urethra. b. Fistulas can form between the bowel and bladder. c. Drink adequate fluids to maintain normal hydration. d. Empty the bladder before and after sexual intercourse.

b. Fistulas can form between the bowel and bladder. Fistulas between the bowel and bladder occur in Crohn's disease and can lead to UTI. Teaching for UTI prevention in general includes good hygiene, adequate fluid intake, and voiding before and after intercourse.

A patient in the emergency department has just been diagnosed with peritonitis caused by a ruptured diverticulum. Which prescribed intervention will the nurse implement first? a. Insert a urinary catheter to drainage. b. Infuse metronidazole (Flagyl) 500 mg IV. c. Send the patient for a computerized tomography scan. d. Place a nasogastric (NG) tube to intermittent low suction.

b. Infuse metronidazole (Flagyl) 500 mg IV. Because peritonitis can be fatal if treatment is delayed, the initial action should be to start antibiotic therapy (after any ordered cultures are obtained). The other actions can be done after antibiotic therapy is initiated.

Which information will the nurse teach a patient with lactose intolerance? a. Ice cream is relatively low in lactose. b. Live-culture yogurt is usually tolerated. c. Heating milk will break down the lactose. d. Nonfat milk is tolerated better than whole milk.

b. Live-culture yogurt is usually tolerated. Lactose-intolerant individuals can usually eat yogurt without experiencing discomfort. Ice cream, nonfat milk, and milk that has been heated are all high in lactose.

The nurse preparing for the annual physical exam of a 50-yr-old man will plan to teach the patient about a. endoscopy. b. colonoscopy. c. computerized tomography screening. d. carcinoembryonic antigen (CEA) testing.

b. colonoscopy. At age 50 years, individuals with an average risk for colorectal cancer (CRC) should begin screening for CRC. Colonoscopy is the gold standard for CRC screening. The other diagnostic tests are not recommended as part of a routine annual physical exam at age 50 years.

Which nursing action will the nurse include in the plan of care for a 35-yr-old male patient admitted with an exacerbation of inflammatory bowel disease (IBD)? a. Restrict oral fluid intake. b. Monitor stools for blood. c. Ambulate six times daily. d. Increase dietary fiber intake.

b. Monitor stools for blood. Because anemia or hemorrhage may occur with IBD, stools should be assessed for the presence of blood. The other actions would not be appropriate for the patient with IBD. Dietary fiber may increase gastrointestinal motility and exacerbate the diarrhea, severe fatigue is common with IBD exacerbations, and dehydration may occur.

A 58-yr-old patient with blunt abdominal trauma from a motor vehicle crash undergoes peritoneal lavage. If the lavage returns brown fecal drainage, which action will the nurse plan to take next? a. Auscultate the bowel sounds. b. Prepare the patient for surgery. c. Check the patient's oral temperature. d.Obtain information about the accident.

b. Prepare the patient for surgery. Return of brown drainage and fecal material suggests perforation of the bowel and the need for immediate surgery. Auscultation of bowel sounds, checking the temperature, and obtaining information about the accident are appropriate actions, but the priority is to prepare to send the patient for emergency surgery.

A 74-yr-old male patient tells the nurse that growing old causes constipation so he has been using a suppository for constipation every morning. Which action should the nurse take first? a. Encourage the patient to increase oral fluid intake. b. Question the patient about risk factors for constipation. c. Suggest that the patient increase intake of high-fiber foods. d. Teach the patient that a daily bowel movement is unnecessary.

b. Question the patient about risk factors for constipation. The nurse's initial action should be further assessment of the patient for risk factors for constipation and for his usual bowel pattern. The other actions may be appropriate but will be based on the assessment.

A new 19-yr-old male patient has familial adenomatous polyposis (FAP). Which action will the nurse in the gastrointestinal clinic include in the plan of care? a. Obtain blood samples for DNA analysis. b. Schedule the patient for yearly colonoscopy. c. Provide preoperative teaching about total colectomy. d. Discuss lifestyle modifications to decrease cancer risk.

b. Schedule the patient for yearly colonoscopy. Patients with FAP should have annual colonoscopy starting at age 16 years and usually have total colectomy by age 25 years to avoid developing colorectal cancer. DNA analysis is used to make the diagnosis but is not needed now for this patient. Lifestyle modifications will not decrease cancer risk for this patient.

The nurse and a licensed practical/vocational nurse (LPN/LVN) are working together to care for a patient who had an esophagectomy 2 days ago. Which action by the LPN/LVN requires that the nurse intervene? a. The LPN/LVN uses soft swabs to provide oral care. b. The LPN/LVN positions the head of the bed in the flat position. c. The LPN/LVN includes the enteral feeding volume when calculating intake. d. The LPN/LVN encourages the patient to use pain medications before coughing.

b. The LPN/LVN positions the head of the bed in the flat position. The patient's bed should be in Fowler's position to prevent reflux and aspiration of gastric contents. The other actions by the LPN/LVN are appropriate.

Which information obtained by the nurse interviewing a 30-yr-old male patient is most important to communicate to the health care provider? a. The patient has a history of constipation. b. The patient has noticed blood in the stools. c. The patient had an appendectomy at age 27. d. The patient smokes a pack/day of cigarettes.

b. The patient has noticed blood in the stools. Blood in the stools is a possible clinical manifestation of colorectal cancer and requires further assessment by the health care provider. The other patient information will also be communicated to the health care provider, but does not indicate an urgent need for further testing or intervention.

A 22-yr-old female patient with an exacerbation of ulcerative colitis is having 15 to 20 stools daily and has excoriated perianal skin. Which patient behavior indicates that teaching regarding maintenance of skin integrity has been effective? a. The patient uses incontinence briefs to contain loose stools. b. The patient uses witch hazel compresses to soothe irritation. c. The patient asks for antidiarrheal medication after each stool. d. The patient cleans the perianal area with soap after each stool.

b. The patient uses witch hazel compresses to soothe irritation. Witch hazel compresses are suggested to reduce anal irritation and discomfort. Incontinence briefs may trap diarrhea and increase the incidence of skin breakdown. Antidiarrheal medications are not given 15 to 20 times a day. The perianal area should be washed with plain water or pH balanced cleanser after each stool.

A critically ill patient with sepsis is frequently incontinent of watery stools. What action by the nurse will prevent complications associated with ongoing incontinence? a. Apply incontinence briefs. b. Use a fecal management system c. Insert a rectal tube with a drainage bag. d. Assist the patient to a commode frequently.

b. Use a fecal management system Fecal management systems are designed to contain loose stools and can be in place for as long as 4 weeks without causing damage to the rectum or anal sphincters. Although incontinence briefs may be helpful, unless they are changed frequently, they are likely to increase the risk for skin breakdown. Rectal tubes are avoided because of possible damage to the anal sphincter and ulceration of the rectal mucosa. A critically ill patient will not be able to tolerate getting up frequently to use the commode or bathroom.

Which information will the nurse include in teaching a patient who had a proctocolectomy and ileostomy for ulcerative colitis? a. Restrict fluid intake to prevent constant liquid drainage from the stoma. b. Use care when eating high-fiber foods to avoid obstruction of the ileum. c. Irrigate the ileostomy daily to avoid having to wear a drainage appliance. d. Change the pouch every day to prevent leakage of contents onto the skin.

b. Use care when eating high-fiber foods to avoid obstruction of the ileum. High-fiber foods are introduced gradually and should be well chewed to avoid obstruction of the ileostomy. Patients with ileostomies lose the absorption of water in the colon and need to take in increased amounts of fluid. The pouch should be drained frequently but is changed every 5 to 7 days. The drainage from an ileostomy is liquid and continuous, so control by irrigation is not possible.

A patient being admitted with an acute exacerbation of ulcerative colitis reports crampy abdominal pain and passing 15 or more bloody stools a day. The nurse will plan to a. administer IV metoclopramide (Reglan). b. discontinue the patient's oral food intake. c. administer cobalamin (vitamin B12) injections. d. teach the patient about total colectomy surgery.

b. discontinue the patient's oral food intake. An initial therapy for an acute exacerbation of inflammatory bowel disease (IBD) is to rest the bowel by making the patient NPO. Metoclopramide increases peristalsis and will worsen symptoms. Cobalamin (vitamin B12) is absorbed in the ileum, which is not affected by ulcerative colitis. Although total colectomy is needed for some patients, there is no indication that this patient is a candidate.

A 33-yr-old male patient with a gunshot wound to the abdomen undergoes surgery, and a colostomy is formed as shown in the accompanying figure. Which information will be included in patient teaching? a. Stool will be expelled from both stomas. b.This type of colostomy is usually temporary. c. Soft, formed stool can be expected as drainage. d. Irrigations can regulate drainage from the stomas.

b.This type of colostomy is usually temporary. A loop, or double-barrel stoma, is usually temporary. Stool will be expelled from the proximal stoma only. The stool from the transverse colon will be liquid and regulation through irrigations will not be possible.

A 26-yr-old woman is being evaluated for vomiting and abdominal pain. Which question from the nurse will be most useful in determining the cause of the patient's symptoms? a. "What type of foods do you eat?" b. "Is it possible that you are pregnant?" c. "Can you tell me more about the pain?" d. "What is your usual elimination pattern?"

c. "Can you tell me more about the pain?" A complete description of the pain provides clues about the cause of the problem. Although the nurse should ask whether the patient is pregnant to determine whether the patient might have an ectopic pregnancy and before any radiology studies are done, this information is not the most useful in determining the cause of the pain. The usual diet and elimination patterns are less helpful in determining the reason for the patient's symptoms.

After change-of-shift report, which patient should the nurse assess first? a. A 42-yr-old patient who has acute gastritis and ongoing epigastric pain b. A 70-yr-old patient with a hiatal hernia who experiences frequent heartburn c. A 60-yr-old patient with nausea and vomiting who has dry mucosa and lethargy d. 53-yr-old patient who has dumping syndrome after a recent partial gastrectomy

c. A 60-yr-old patient with nausea and vomiting who has dry mucosa and lethargy This patient is at high risk for problems such as aspiration, dehydration, and fluid and electrolyte disturbances. The other patients will also need to be assessed, but the information about them indicates symptoms that are typical for their diagnoses and are not life threatening.

Which care activity for a patient with a paralytic ileus is appropriate for the registered nurse (RN) to delegate to unlicensed assistive personnel (UAP)? a. Auscultation for bowel sounds b. Nasogastric (NG) tube irrigation c. Applying petroleum jelly to the lips d. Assessment of the nares for irritation

c. Applying petroleum jelly to the lips UAP education and scope of practice include patient hygiene such as oral care. The other actions require education and scope of practice appropriate to the RN.

A 72-yr-old male patient with dehydration caused by an exacerbation of ulcerative colitis is receiving 5% dextrose in normal saline at 125 mL/hour. Which assessment finding by the nurse is most important to report to the health care provider? a. Patient has not voided for the last 4 hours. b. Skin is dry with poor turgor on all extremities. c. Crackles are heard halfway up the posterior chest. d. Patient has had 5 loose stools over the previous 6 hours.

c. Crackles are heard halfway up the posterior chest. The presence of crackles in an older patient receiving IV fluids at a high rate suggests volume overload and a need to reduce the rate of the IV infusion. The other data will also be reported but are consistent with the patient's age and diagnosis and do not require a change in the prescribed treatment.

Which activity in the care of a patient with a new colostomy could the nurse delegate to unlicensed assistive personnel (UAP)? a. Document the appearance of the stoma. b. Place a pouching system over the ostomy. c. Drain and measure the output from the ostomy. d. Check the skin around the stoma for breakdown.

c. Drain and measure the output from the ostomy. Draining and measuring the output from the ostomy is included in UAP education and scope of practice. The other actions should be implemented by LPNs or RNs.

A patient is admitted to the emergency department with severe abdominal pain and rebound tenderness. Vital signs include temperature 102°F (38.3°C), pulse 120 beats/min, respirations 32 breaths/min, and blood pressure (BP) 82/54 mm Hg. Which prescribed intervention should the nurse implement first? a. Administer IV ketorolac 15 mg for pain relief. b. Draw a blood sample for a complete blood count (CBC). c. Infuse a liter of lactated Ringer's solution over 30 minutes. d. Send the patient for an abdominal computed tomography (CT) scan.

c. Infuse a liter of lactated Ringer's solution over 30 minutes. The priority for this patient is to treat the patient's hypovolemic shock with fluid infusion. The other actions should be implemented after starting the fluid infusion.

A 76-yr-old patient with obstipation has a fecal impaction and is incontinent of liquid stool. Which action should the nurse take first? a. Administer bulk-forming laxatives. b. Assist the patient to sit on the toilet. c. Manually remove the impacted stool. d. Increase the patient's oral fluid intake.

c. Manually remove the impacted stool. The initial action with a fecal impaction is manual disimpaction. The other actions will be used to prevent future constipation and impactions.

Which diet choice by the patient with an acute exacerbation of inflammatory bowel disease (IBD) indicates a need for more teaching? a. Scrambled eggs b. White toast and jam c. Oatmeal with cream d. Pancakes with syrup

c. Oatmeal with cream During acute exacerbations of IBD, the patient should avoid high-fiber foods such as whole grains. High-fat foods also may cause diarrhea in some patients. The other choices are low residue and would be appropriate for this patient.

Which action will the nurse include in the plan of care for a patient who is being admitted with Clostridium difficile? a. Teach the patient about proper food storage. b. Order a diet without dairy products for the patient. c. Place the patient in a private room on contact isolation. d. Teach the patient about why antibiotics will not be used.

c. Place the patient in a private room on contact isolation. Because C. difficile is highly contagious, the patient should be placed in a private room, and contact precautions should be used. There is no need to restrict dairy products for this type of diarrhea. Metronidazole (Flagyl) is frequently used to treat C. difficile infections. Improper food handling and storage do not cause C. difficile.

A young adult patient is admitted to the hospital for evaluation of right lower quadrant abdominal pain with nausea and vomiting. Which action should the nurse take? a. Assist the patient to cough and deep breathe. b. Palpate the abdomen for rebound tenderness. c. Suggest the patient lie on the side, flexing the right leg. d. Encourage the patient to sip clear, noncarbonated liquids.

c. Suggest the patient lie on the side, flexing the right leg. The patient's clinical manifestations are consistent with appendicitis. Lying still with the right leg flexed is often the most comfortable position. Checking for rebound tenderness frequently is unnecessary and uncomfortable for the patient. The patient should be NPO in case immediate surgery is needed. The patient will need to know how to cough and deep breathe postoperatively, but coughing will increase pain at this time.

After a patient has had a hemorrhoidectomy at an outpatient surgical center, which instructions will the nurse include in discharge teaching? a. Maintain a low-residue diet until the surgical area is healed. b. Use ice packs on the perianal area to relieve pain and swelling. c. Take prescribed pain medications before you expect a bowel movement. d. Delay having a bowel movement for several days until you are well healed.

c. Take prescribed pain medications before you expect a bowel movement. Bowel movements may be very painful, and patients may avoid defecation unless pain medication is taken before the bowel movement. A high-residue diet will increase stool bulk and prevent constipation. Delay of bowel movements is likely to lead to constipation. Warm sitz baths rather than ice packs are used to relieve pain and keep the surgical area clean.

After several days of antibiotic therapy, an older hospitalized patient develops watery diarrhea. Which action should the nurse take first? a. Notify the health care provider. b. Obtain a stool specimen for analysis. c. Teach the patient about handwashing. d. Place the patient on contact precautions.

c. Teach the patient about handwashing. The patient's history and new onset diarrhea suggest a C. difficile infection, which requires implementation of contact precautions to prevent spread of the infection to other patients. The other actions are also appropriate but can be accomplished after contact precautions are implemented.

A 19-yr-old woman is brought to the emergency department with a knife handle protruding from her abdomen. During the initial assessment of the patient, the nurse should a. remove the knife and assess the wound. b. determine the presence of Rovsing sign. c. check for circulation and tissue perfusion. d. insert a urinary catheter and assess for hematuria.

c. check for circulation and tissue perfusion. The initial assessment is focused on determining whether the patient has hypovolemic shock. The knife should not be removed until the patient is in surgery, where bleeding can be controlled. Rovsing sign is assessed in the patient with suspected appendicitis. Assessment for bladder trauma is not part of the initial assessment.

Four hours after a bowel resection, a 74-yr-old male patient with a nasogastric tube to suction complains of nausea and abdominal distention. The first action by the nurse should be to a. auscultate for hypotonic bowel sounds. b. notify the patient's health care provider. c. check for tube placement and reposition it. d. remove the tube and replace it with a new one.

c. check for tube placement and reposition it. Repositioning the tube will frequently facilitate drainage. Because this is a common occurrence, it is not appropriate to notify the health care provider unless other interventions do not resolve the problem. Information about the presence or absence of bowel sounds will not be helpful in improving drainage. Removing the tube and replacing it are unnecessarily traumatic to the patient, so that would only be done if the tube was completely occluded.

Which question from the nurse would help determine if a patient's abdominal pain might indicate irritable bowel syndrome (IBS)? a. "Have you been passing a lot of gas?" b. "What foods affect your bowel patterns?" c. "Do you have any abdominal distention?" d. "How long have you had abdominal pain?"

d. "How long have you had abdominal pain?" One criterion for the diagnosis of irritable bowel syndrome is the presence of abdominal discomfort or pain for at least 3 months. Abdominal distention, flatulence, and food intolerance are associated with IBS but are not diagnostic criteria.

Which patient should the nurse assess first after receiving change-of-shift report? a. A 60-yr-old patient whose new ileostomy has drained 800 mL over the previous 8 hours b. A 50-yr-old patient with familial adenomatous polyposis who has occult blood in the stool c. A 40-yr-old patient with ulcerative colitis who has had six liquid stools in the previous 4 hours d. A 30-yr-old patient who has abdominal distention and an apical heart rate of 136 beats/minute

d. A 30-yr-old patient who has abdominal distention and an apical heart rate of 136 beats/minute The patient's abdominal distention and tachycardia suggest hypovolemic shock caused by problems such as peritonitis or intestinal obstruction, which will require rapid intervention. The other patients should also be assessed as quickly as possible, but the data do not indicate any life-threatening complications associated with their diagnoses.

A 40-yr-old male patient has had a herniorrhaphy to repair an incarcerated inguinal hernia. Which patient teaching will the nurse provide before discharge? a. Soak in sitz baths several times each day. b. Cough 5 times each hour for the next 48 hours. c. Avoid use of acetaminophen (Tylenol) for pain. d. Apply a scrotal support and ice to reduce swelling.

d. Apply a scrotal support and ice to reduce swelling. A scrotal support and ice are used to reduce edema and pain. Coughing will increase pressure on the incision. Sitz baths will not relieve pain and would not be of use after this surgery. Acetaminophen can be used for postoperative pain.

Which breakfast choice indicates a patient's good understanding of information about a diet for celiac disease? a. Oatmeal with nonfat milk b. wheat toast with butter c. Bagel with low-fat cream cheese d. Corn tortilla with scrambled eggs

d. Corn tortilla with scrambled eggs Avoidance of gluten-containing foods is the only treatment for celiac disease. Corn does not contain gluten, but oatmeal and wheat do.

A patient who has chronic constipation asks the nurse about the use of psyllium (Metamucil). Which information will the nurse include in the response? a. Absorption of fat-soluble vitamins may be reduced by fiber-containing laxatives. b. Dietary sources of fiber should be eliminated to prevent excessive gas formation. c. Use of this type of laxative to prevent constipation does not cause adverse effects. d. Large amounts of fluid should be taken to prevent impaction or bowel obstruction.

d. Large amounts of fluid should be taken to prevent impaction or bowel obstruction. A high fluid intake is needed when patients are using bulk-forming laxatives to avoid worsening constipation. Although bulk-forming laxatives are generally safe, the nurse should emphasize the possibility of constipation or obstipation if inadequate fluid intake occurs. Although increased gas formation is likely to occur with increased dietary fiber, the patient should gradually increase dietary fiber and eventually may not need the psyllium. Fat-soluble vitamin absorption is blocked by stool softeners and lubricants, not by bulk-forming laxatives.

A patient with diverticulosis has a large bowel obstruction. The nurse will monitor for a. referred back pain. b. metabolic alkalosis. c. projectile vomiting. d. abdominal distention.

d. abdominal distention. Abdominal distention is seen in lower intestinal obstruction. Referred back pain is not a common clinical manifestation of intestinal obstruction. Metabolic alkalosis is common in high intestinal obstruction because of the loss of HCl acid from vomiting. Projectile vomiting is associated with higher intestinal obstruction.

The nurse will plan to teach a patient with Crohn's disease who has megaloblastic anemia about the need for a. iron dextran infusions b. oral ferrous sulfate tablets. c. routine blood transfusions. d. cobalamin (B12) supplements.

d. cobalamin (B12) supplements. Crohn's disease frequently affects the ileum, where absorption of cobalamin occurs. Cobalamin must be administered regularly by nasal spray or IM to correct the anemia. Iron deficiency does not cause megaloblastic anemia. The patient may need occasional transfusions but not regularly scheduled transfusions.

The nurse is providing preoperative teaching for a patient scheduled for an abdominal-perineal resection. Which information will the nurse include? a. The patient will begin sitting in a chair at the bedside on the first postoperative day. b. IV antibiotics will be started at least 24 hours before surgery to reduce the bowel bacteria. c. An additional surgery in 8 to 12 weeks will be used to create an ileal-anal reservoir. d. document stoma assessment findings.

d. document stoma assessment findings. The stoma appearance indicates good circulation to the stoma. There is no indication that surgical intervention is needed or that frequent stoma monitoring is required. Swelling of the stoma is normal for 2 to 3 weeks after surgery, and an ice pack is not needed.

The nurse will anticipate teaching a patient experiencing frequent heartburn about a. a barium swallow. b. radionuclide tests. c. endoscopy procedures. d. proton pump inhibitors.

d. proton pump inhibitors. Because diagnostic testing for heartburn that is probably caused by gastroesophageal reflux disease (GERD) is expensive and uncomfortable, proton pump inhibitors are frequently used for a short period as the first step in the diagnosis of GERD. The other tests may be used but are not usually the first step in diagnosis.

When a patient is diagnosed with achalasia, the nurse will teach the patient that a. lying down after meals is recommended. b. a liquid or blenderized diet will be necessary. c. drinking fluids with meals should be avoided. d. treatment may include endoscopic procedures.

d. treatment may include endoscopic procedures. Endoscopic and laparoscopic procedures are the most effective therapy for improving symptoms caused by achalasia. Keeping the head elevated after eating will improve esophageal emptying. A semisoft diet is recommended to improve esophageal emptying. Patients are advised to drink fluid with meals.

The nurse explaining esomeprazole (Nexium) to a patient with recurring heartburn describes that the medication a. reduces gastroesophageal reflux by increasing the rate of gastric emptying. b. neutralizes stomach acid and provides relief of symptoms in a few minutes. c. coats and protects the lining of the stomach and esophagus from gastric acid. d. treats gastroesophageal reflux disease by decreasing stomach acid production.

d. treats gastroesophageal reflux disease by decreasing stomach acid production. The proton pump inhibitors decrease the rate of gastric acid secretion. Promotility drugs such as metoclopramide (Reglan) increase the rate of gastric emptying. Cryoprotective medications such as sucralfate (Carafate) protect the stomach. Antacids neutralize stomach acid and work rapidly.

The results of a patient's recent endoscopy indicate the presence of peptic ulcer disease (PUD). Which teaching point should the nurse provide to the patient based on this new diagnosis? "It would be beneficial for you to stop drinking alcohol." "You'll need to drink at least 2 to 3 glasses of milk daily." "Many people find that a minced or pureed diet eases their symptoms of PUD." "You can keep your present diet and minimize symptoms by taking medication."

"It would be beneficial for you to stop drinking alcohol." Alcohol increases the amount of stomach acid produced, so it should be avoided. Although there is no specific recommended dietary modification for PUD, most patients find it necessary to make some dietary modifications to minimize symptoms. Milk may worsen PUD.

A patient is scheduled for a percutaneous transhepatic cholangiography to restore biliary drainage. The nurse discusses the patient's health history and is most concerned if the patient makes which statement? "I am allergic to bee stings." "My tongue swells when I eat shrimp." "I have had epigastric pain for 2 months." "I have a pacemaker because my heart rate was slow."

"My tongue swells when I eat shrimp." The percutaneous transhepatic cholangiography procedure will include the use of radiopaque contrast medium. Patients allergic to shellfish and iodine are also allergic to contrast medium.

The nurse teaches older adults at a community center how to prevent food poisoning at social events. Which community member statement reflects accurate understanding? "Pasteurized juices and milk are safe to drink." "Raw cookie dough is safe to eat if it is cold." "Fresh fruits do not need washed before eating." "Ground beef is safe to eat if it is slightly pink."

"Pasteurized juices and milk are safe to drink." Drink only pasteurized milk, juice, or cider. Ground beef should be cooked thoroughly. Browned meat can still harbor live bacteria. Cook ground beef until a thermometer reads at least 160° F. If a thermometer is unavailable, decrease the risk of illness by cooking the ground beef until there is no pink color in the middle. Fruits and vegetables should be washed thoroughly, especially those that will not be cooked. Do not eat raw food products, such as dough, that are supposed to be cooked.

Which patient statement indicates that the nurse's postoperative teaching after a gastroduodenostomy has been effective? a. "I will drink more liquids with my meals." b. "I should choose high carbohydrate foods." c. "Vitamin supplements may prevent anemia." d. "Persistent heartburn is common after surgery."

"Vitamin supplements may prevent anemia." Cobalamin deficiency may occur after partial gastrectomy, and the patient may need to receive cobalamin via injections or nasal spray. Although peptic ulcer disease may recur, persistent heartburn is not expected after surgery, and the patient should call the health care provider if this occurs. Ingestion of liquids with meals is avoided to prevent dumping syndrome. Foods that have moderate fat and low carbohydrate should be chosen to prevent dumping syndrome.

The nurse is teaching a group of college students how to prevent food poisoning. Which comment shows an understanding of foodborne illness protection? "To save refrigerator space, leftover food can be kept on the counter if it is in a sealed container." "Eating raw cookie dough from the package is a great snack when you do not have time to bake." "Since we only have one cutting board, we can cut up chicken and salad vegetables at the same time." "When the cafeteria gave me a pink hamburger, I sent it back and asked for a new bun and clean plate."

"When the cafeteria gave me a pink hamburger, I sent it back and asked for a new bun and clean plate." The student who did not accept the pink hamburger and asked for a new bun and clean plate understood that the pink meat may not have reached 160° F and could be contaminated with bacteria. Improperly storing cooked foods, eating raw cookie dough from a refrigerated package, and only using one cutting board without washing it with hot soapy water between the chicken and salad vegetables could all lead to food poisoning from contamination.

A patient was involved in a motor vehicle crash and reports an inability to have a bowel movement. What is the best response by the nurse? "Your parasympathetic nervous system is now working to slow the GI tract." "The circulation in the GI system has been increased, so less waste is removed." "Your sympathetic nervous system was activated, so there is slowing of the GI tract." "You may have bruised your intestines, so no stool will be produced for a few days."

"Your sympathetic nervous system was activated, so there is slowing of the GI tract." The constipation is most likely related to the sympathetic nervous system activation from the stress related to the accident. Sympathetic nervous system activation can decrease peristalsis. Even without oral intake for a short time, stool will be formed. The parasympathetic system stimulates peristalsis. The circulation to the gastrointestinal system is decreased with stress.

Which patient is at highest risk for developing oral candidiasis? A 74-yr-old patient who has vitamin B and C deficiencies A 22-yr-old patient who smokes 2 packs of cigarettes per day A 58-yr-old patient who is receiving amphotericin B for 2 days. A 32-yr-old patient who is receiving ciprofloxacin for 3 weeks.

A 32-yr-old patient who is receiving ciprofloxacin for 3 weeks. Oral candidiasis is caused by prolonged antibiotic treatment (e.g., ciprofloxacin) or high doses of corticosteroids. Amphotericin B is used to treat candidiasis. Vitamin B and C deficiencies may lead to Vincent's infection. Use of tobacco products leads to stomatitis, not candidiasis.

When caring for the patient with heart failure, the nurse knows that which gastrointestinal process is most dependent on cardiac output and may affect the patient's nutritional status? a. Ingestion b. Digestion c. Absorption d. Elimination

Absorption Substances that interface with the absorptive surfaces of the gastrointestinal tract (primarily in the small intestine) diffuse across the intestinal membranes into intestinal capillaries and are then carried to other parts of the body for use in energy production. The cardiac output provides the blood flow for this absorption of nutrients to occur.

Question 18 of 18 The nurse is reviewing the home medication list for a patient admitted with suspected hepatic failure. Which medication could cause hepatotoxicity? Digoxin Nitroglycerin Ciprofloxacin Acetaminophen

Acetaminophen Many chemicals and drugs are potentially hepatotoxic and result in significant patient harm unless monitored closely. For example, chronic high doses of acetaminophen and nonsteroidal antiinflammatory drugs may be hepatotoxic.

A patient has peptic ulcer disease that has been associated with Helicobacter pylori. About which medications will the nurse plan to teach the patient? a. Sucralfate (Carafate), nystatin, and bismuth (Pepto-Bismol) b. Metoclopramide (Reglan), bethanechol (Urecholine), and promethazine c. Amoxicillin (Amoxil), clarithromycin (Biaxin), and omeprazole (Prilosec) d. Famotidine (Pepcid), magnesium hydroxide (Mylanta), and pantoprazole (Protonix)

Amoxicillin (Amoxil), clarithromycin (Biaxin), and omeprazole (Prilosec) The drugs used in triple drug therapy include a proton pump inhibitor such as omeprazole and the antibiotics amoxicillin and clarithromycin. The other combinations listed are not included in the protocol for H. pylori infection.

The patient with chronic gastritis is being put on a combination of medications to eradicate Helicobacter pylori. Which drugs does the nurse know will probably be used? Antibiotic(s), antacid, and corticosteroid Antibiotic(s), aspirin, and antiulcer/protectant Antibiotic(s), proton pump inhibitor, and bismuth Antibiotic(s) and nonsteroidal antiinflammatory drugs (NSAIDs)

Antibiotic(s), proton pump inhibitor, and bismuth To eradicate H. pylori, a combination of antibiotics, a proton pump inhibitor, and possibly bismuth (for quadruple therapy) will be used. Corticosteroids, aspirin, and NSAIDs are drugs that can cause gastritis and do not affect H. pylori.

The nurse is caring for a postoperative patient who has just vomited yellow-green liquid. Which action would be an appropriate nursing intervention? Offer the patient an herbal supplement such as ginseng. Discontinue medications that may cause nausea or vomiting. Apply a cool washcloth to the forehead and provide mouth care. Take the patient for a walk in the hallway to promote peristalsis.

Apply a cool washcloth to the forehead and provide mouth care. Cleansing the face and hands with a cool washcloth and providing mouth care are appropriate comfort interventions for nausea and vomiting. Ginseng is not used to treat postoperative nausea and vomiting. Unnecessary activity should be avoided. The patient should rest in a quiet environment. Medications may be temporarily held until the acute phase is over, but the medications should not be discontinued without consultation with the health care provider.

The nurse is performing an abdominal assessment for a patient. Which assessment technique by the nurse is most accurate? Palpate the abdomen before auscultation. Percuss the abdomen before auscultation. Auscultate the abdomen before palpation. Perform deep palpation before light palpation.

Auscultate the abdomen before palpation. During examination of the abdomen, auscultation is done before percussion and palpation because these latter procedures may alter the bowel sounds.

Inspection of an older patient's mouth reveals the presence of white, curd-like lesions on the patient's tongue. What does the nurse recognize is the most likely reason for this abnormal assessment finding? Herpesvirus Candida albicans Vitamin deficiency Irritation from ill-fitting dentures

Candida albicans White, curd-like lesions surrounded by erythematous mucosa are associated with oral candidiasis. Herpesvirus causes benign vesicular lesions in the mouth. Vitamin deficiencies may cause a reddened, ulcerated, swollen tongue. Irritation from ill-fitting dentures will cause friable, edematous, painful, bleeding gingivae.

Question 16 of 18 A patient with abdominal pain is being prepared for surgery to make an incision into the common bile duct to remove stones. What procedure will the nurse prepare the patient for? a. Colectomy b. Cholecystectomy c. Choledocholithotomy d. Choledochojejunostomy

Choledocholithotomy A choledocholithotomy is an opening into the common bile duct for the removal of stones. A colectomy is the removal of the colon. The cholecystectomy is the removal of the gallbladder. The choledochojejunostomy is an opening between the common bile duct and jejunum.

Which item should the nurse offer to the patient who is to restart oral intake after being NPO due to nausea and vomiting? a. Glass of orange juice b. Dish of lemon gelatin c. Cup of coffee with cream d. Bowl of hot chicken broth

Clear cool liquids are usually the first foods started after a patient has been nauseated. Acidic foods such as orange juice, very hot foods, and coffee are poorly tolerated when patients have been nauseated.

The nurse determines that teaching regarding cobalamin injections has been effective when the patient with chronic atrophic gastritis states a. "The cobalamin injections will prevent gastric inflammation." b. "The cobalamin injections will prevent me from becoming anemic." c. "These injections will increase the hydrochloric acid in my stomach." d. "These injections will decrease my risk for developing stomach cancer."

Cobalamin supplementation prevents the development of pernicious anemia. Chronic gastritis may cause achlorhydria, but cobalamin does not correct this. The loss of intrinsic factor secretion with chronic gastritis is permanent, and the patient will need lifelong supplementation with cobalamin. The incidence of stomach cancer is higher in patients with chronic gastritis, but cobalamin does not reduce the risk for stomach cancer.

The nurse should recognize that the liver performs which functions? (Select all that apply.) Bile storage Detoxification Protein metabolism Steroid metabolism Red blood cell (RBC) production

Detoxification Protein metabolism Steroid metabolism The liver performs multiple major functions that aid in the maintenance of homeostasis. These include metabolism of proteins and steroids as well as detoxification of drugs and metabolic waste products. The Kupffer cells of the liver participate in the breakdown of old RBCs. The liver produces bile, but storage occurs in the gallblader

After administering a dose of promethazine to a patient with nausea and vomiting, what medication side effect does the nurse explain is common and expected? Tinnitus Drowsiness Reduced hearing Sensation of falling

Drowsiness Although being given to this patient as an antiemetic, promethazine also has sedative and amnesic properties. For this reason, the patient is likely to experience drowsiness as an adverse effect of the medication. Tinnitus, reduced hearing, and loss of balance are not side effects of promethazine.

The nurse is caring for a patient being treated with IV fluid therapy for severe vomiting. As the patient recovers and begins to tolerate oral intake, which food choice would be most appropriate? Iced tea Dry toast Hot coffee Plain yogurt

Dry toast Dry toast or crackers may alleviate the feeling of nausea and prevent further vomiting. Water is the initial fluid of choice. Extremely hot or cold liquids and fatty foods are generally not well tolerated.

A patient who had a gastroduodenostomy (Billroth I operation) for stomach cancer reports generalized weakness, sweating, palpitations, and dizziness 15 to 30 minutes after eating. What long-term complication does the nurse suspect is occurring? Malnutrition Bile reflux gastritis Dumping syndrome Postprandial hypoglycemia

Dumping syndrome After a Billroth I operation, dumping syndrome may occur 15 to 30 minutes after eating because of the hypertonic fluid going to the intestine and additional fluid being drawn into the bowel. Malnutrition may occur but does not cause these symptoms. Bile reflux gastritis cannot happen when the stomach has been removed. Postprandial hypoglycemia occurs with similar symptoms, but 2 hours after eating.

A patient is scheduled for surgery with general anesthesia in 1 hour and is observed with a moist but empty water glass in his hand. Which assessment finding may indicate that the patient drank a glass of water? Easily heard, loud gurgling in abdomen High-pitched, hollow sounds in abdomen Flat abdomen without movement upon inspection Tenderness in left upper quadrant upon palpation

Easily heard, loud gurgling in abdomen If the patient drank water on an empty stomach, gurgling can be assessed without a stethoscope or assessed with auscultation. High-pitched, hollow sounds are tympanic and indicate an empty cavity. A flat abdomen and tenderness do not indicate that the patient drank a glass of water.

A patient is seeking emergency care after choking on a piece of steak. The nursing assessment reveals a history of alcohol use, cigarette smoking, and hemoptysis. Which diagnostic study is most likely to be performed on this patient? Barium swallow Endoscopic biopsy Capsule endoscopy Endoscopic ultrasonography

Endoscopic biopsy Because of this patient's history of alcohol use, smoking, and hemoptysis and the current choking episode, cancer may be present. A biopsy is necessary to make a definitive diagnosis of cancer, so an endoscope will be used to obtain a biopsy and observe other abnormalities as well. A barium swallow may show narrowing of the esophagus, but it is more diagnostic for achalasia. An endoscopic ultrasonography may be used to stage esophageal cancer. Capsule endoscopy can show esophageal problems but is more often used for small intestine problems. A barium swallow, capsule endoscopy, and endoscopic ultrasonography cannot provide a definitive diagnosis for cancer.

The nurse is preparing a patient for a capsule endoscopy. What should the nurse ensure is included in the preparation? Ensure the patient understands the required bowel preparation. Have the patient return to the procedure room for removal of the capsule. Teach the patient to maintain a clear liquid diet throughout the procedure. Explain to the patient that conscious sedation will be used during capsule placement.

Ensure the patient understands the required bowel preparation. A capsule endoscopy study involves the patient performing a bowel prep to cleanse the bowel before swallowing the capsule. The patient will be on a clear liquid diet for 1 to 2 days before the procedure and will remain NPO for 4 to 6 hours after swallowing the capsule. The capsule is disposable and will pass naturally with the bowel movement, although the monitoring device will need to be removed.

The nurse determines a patient has experienced the beneficial effects of famotidine when which symptom is relieved? Nausea Belching Epigastric pain Difficulty swallowing

Epigastric pain Famotidine is an H2-receptor antagonist that inhibits parietal cell output of HCl acid and minimizes damage to gastric mucosa related to hyperacidity, thus relieving epigastric pain. It is not indicated for nausea, belching, and dysphagia.

A patient with a family history of adenomatous polyposis had a colonoscopy with removal of multiple polyps. After the procedure, which signs and symptoms should the nurse teach the patient to report immediately? Fever and abdominal pain Flatulence and liquid stool Loudly audible bowel sounds Sleepiness and abdominal cramps

Fever and abdominal pain The patient should be taught to observe for signs of rectal bleeding and peritonitis. Fever, malaise, and abdominal pain and distention could indicate a perforated bowel with peritonitis.

A patient had a gastric resection for stomach cancer. The nurse plans to teach the patient about decreased secretion of which hormone? Gastrin Secretin Cholecystokinin Gastric inhibitory peptide

Gastrin Gastrin is the hormone activated in the stomach (and duodenal mucosa) by stomach distention that stimulates gastric acid secretion and motility and maintains lower esophageal sphincter tone. Secretin, cholecystokinin, and gastric inhibitory peptide are all secreted from the duodenal mucosa.

A patient has a sliding hiatal hernia. What priority nursing intervention will reduce the symptoms of heartburn and dyspepsia? Keeping the patient NPO Putting the bed in the Trendelenburg position Having the patient eat 4 to 6 smaller meals each day Giving various antacids to determine which one works for the patient

Having the patient eat 4 to 6 smaller meals each day Eating smaller meals during the day will decrease the gastric pressure and symptoms of hiatal hernia. Keeping the patient NPO or in a Trendelenburg position is not safe or realistic for a long period of time for any patient. Varying antacids will only be done with the health care provider's prescription, so this is not a nursing intervention.

The patient with a history of irritable bowel disease and gastroesophageal reflux disease (GERD) is admitted with a diagnosis of diverticulitis and has received a dose of Mylanta 30 mL PO. The nurse will determine the medication was effective when which symptom has been resolved? Diarrhea Heartburn Constipation Lower abdominal pain

Heartburn Mylanta is an antacid that contains both aluminum and magnesium. It is indicated for the relief of gastrointestinal discomfort, such as heartburn associated with GERD. Mylanta can cause both diarrhea and constipation as a side effect. Mylanta does not affect lower abdominal pain.

The nurse is assessing a patient admitted with a possible bowel obstruction. Which assessment finding would be expected in this patient? Tympany to abdominal percussion Aortic pulsation visible in epigastric region High-pitched sounds on abdominal auscultation Liver border palpable 1 cm below the right costal margin

High-pitched sounds on abdominal auscultation The bowel sounds are higher pitched (rushes and tinkling) when the intestines are under tension, as in intestinal obstruction. Bowel sounds may also be diminished or absent with an intestinal obstruction. Normal findings include aortic pulsations on inspection and tympany with percussion, and the liver may be palpable 1 to 2 cm along the right costal margin.

The patient is having an esophagoenterostomy with anastomosis of a segment of the colon to replace the resected portion. What initial postoperative care should the nurse expect when this patient returns to the nursing unit? Deep breathe, cough, and use spirometer every 4 hours. Maintain an upright position for at least 2 hours after eating. NG will have bloody drainage and it should not be repositioned. Keep in a supine position to prevent movement of the anastomosis.

NG will have bloody drainage and it should not be repositioned. The patient will have bloody drainage from the nasogastric (NG) tube for 8 to 12 hours, and it should not be repositioned or reinserted without contacting the surgeon. Deep breathing and spirometry will be done every 2 hours. Coughing would put too much pressure in the area and should not be done. Because the patient will have the NG tube, the patient will not be eating yet. The patient should be kept in a semi-Fowler's or Fowler's position, not supine, to prevent reflux and aspiration of secretions.

A 74-year-old female patient with osteoporosis is diagnosed with gastroesophageal reflux disease (GERD). Which over-the-counter medication to treat GERD should be used with caution? Sucralfate Cimetidine Omeprazole Metoclopramide

Omeprazole There is a potential link between proton pump inhibitors (PPIs) (e.g., omeprazole) use and bone metabolism. Long-term use or high doses of PPIs may increase the risk of fractures of the hip, wrist, and spine.

The patient receiving chemotherapy rings the call bell and reports the onset of nausea. The nurse should prepare an as-needed dose of which medication? Zolpidem Ondansetron Dexamethasone Morphine sulfate

Ondansetron Ondansetron is a 5-HT3 receptor antagonist antiemetic that is especially effective in reducing cancer chemotherapy-induced nausea and vomiting. Morphine sulfate may cause nausea and vomiting. Zolpidem does not relieve nausea and vomiting. Dexamethasone is usually used in combination with ondansetron for acute and chemotherapy-induced emesis.

The nurse is caring for a patient who reports abdominal pain and hematemesis. Which new assessment finding(s) would indicate the patient's condition is declining? Pallor and diaphoresis Reddened peripheral IV site Guaiac-positive diarrhea stools Heart rate 90, respiratory rate 20, BP 110/60

Pallor and diaphoresis A patient with hematemesis has some degree of bleeding from an unknown source. Guaiac-positive diarrhea stools would be an expected finding. When monitoring the patient for stability, the nurse observes for signs of hypovolemic shock such as tachycardia, tachypnea, hypotension, altered level of consciousness, pallor, and cool and clammy skin. A reddened peripheral IV site will require assessment to determine the need for reinsertion. Access would be critical in the immediate treatment of shock, but the IV site does not represent a decline in condition.

A patient reports severe pain when the nurse assesses for rebound tenderness. What may this assessment finding indicate? Hepatic cirrhosis Hypersplenomegaly Gallbladder distention Peritoneal inflammation

Peritoneal inflammation When palpating for rebound tenderness, the problem area of the abdomen will produce pain and severe muscle spasm when there is peritoneal inflammation. Hepatic cirrhosis, hypersplenomegaly, and gallbladder distention do not manifest with rebound tenderness.

A patient with a history of peptic ulcer disease presents to the emergency department with severe abdominal pain and a rigid, boardlike abdomen. The health care provider suspects a perforated ulcer. Which interventions should the nurse anticipate? Providing IV fluids and inserting a nasogastric (NG) tube Administering oral bicarbonate and testing the patient's gastric pH level Performing a fecal occult blood test and administering IV calcium gluconate Starting parenteral nutrition and placing the patient in a high Fowler's position

Providing IV fluids and inserting a nasogastric (NG) tube A perforated peptic ulcer requires IV replacement of fluid losses and continued gastric aspiration by NG tube. Nothing is given by mouth, and gastric pH testing is not a priority. Calcium gluconate is not a medication directly relevant to the patient's suspected diagnosis, and parenteral nutrition is not a priority in the short term.

After receiving a dose of metoclopramide, which patient assessment finding would indicate the medication was effective? Decreased blood pressure Absence of muscle tremors Relief of nausea and vomiting No further episodes of diarrhea

Relief of nausea and vomiting Metoclopramide is classified as a prokinetic and antiemetic medication. If it is effective, the patient's nausea and vomiting should resolve. Metoclopramide does not affect blood pressure, muscle tremors, or diarrhea.

The nurse is performing a focused abdominal assessment of a patient who has been recently admitted. In order to palpate the patient's liver, where should the nurse palpate the patient's abdomen? a. Left lower quadrant b. Left upper quadrant c. Right lower quadrant d. Right upper quadrant

Right upper quadrant Although the left lobe of the liver is located in the left upper quadrant of the abdomen, the bulk of the liver is located in the right upper quadrant.

A patient was admitted with epigastric pain because of a gastric ulcer. Which patient assessment warrants an urgent change in the nursing plan of care? Back pain 3 or 4 hours after eating a meal Chest pain relieved with eating or drinking water Burning epigastric pain 90 minutes after breakfast Rigid abdomen and vomiting following indigestion

Rigid abdomen and vomiting following indigestion A rigid abdomen with vomiting in a patient who has a gastric ulcer indicates a perforation of the ulcer, especially if the manifestations of perforation appear suddenly. Midepigastric pain is relieved by eating, drinking water, or antacids with duodenal ulcers, not gastric ulcers. Back pain 3 to 4 hours after a meal is more likely to occur with a duodenal ulcer. Burning epigastric pain 1 to 2 hours after a meal is an expected manifestation of a gastric ulcer related to increased gastric secretions and does not cause an urgent change in the nursing plan of care.

A patient is suspected of having acute pancreatitis after presenting to the emergency department with severe abdominal pain. Which laboratory result would indicate the presence of acute pancreatitis? Gastric pH of 1.4 Blood glucose of 104 Serum amylase of 820 U/L Serum potassium of 3.5 mEq/L

Serum amylase of 820 U/L Elevated serum amylase levels indicate early pancreatic dysfunction and are used to diagnose acute pancreatitis. The normal serum amylase levels range from 28 to 85 U/L. Serum lipase levels stay elevated longer than serum amylase in acute pancreatitis. Blood glucose, gastric pH, and potassium levels are not direct indicators of acute pancreatic dysfunction.

A patient reporting nausea receives a dose of metoclopramide. Which potential adverse effect should the nurse tell the patient to report? Tremors Constipation Double vision Numbness in fingers and toes

Tremors Extrapyramidal side effects, including tremors and tardive dyskinesias, may occur with metoclopramide administration. Constipation, double vision, and numbness in fingers and toes are not adverse effects of metoclopramide.

A patient with oral cancer is not eating. A small-bore feeding tube was inserted, and the patient started on enteral feedings. Which patient goal would best indicate improvement? a. Weight gain of 1 kg in 1 week b. Tolerated the tube feeding without nausea c. Consumed 50% of clear liquid tray this shift d. The feeding tube remained in proper placement

Weight gain of 1 kg in 1 week The best goal for a patient with oral cancer that is not eating would be to note weight gain rather than loss. Consuming 50% of the clear liquid tray is not a realistic goal. The absence of nausea and proper tube placement, while desired, do not indicate nutritional improvement.

An older adult patient is seen in the primary care provider's office for a well check reports difficulty swallowing. What common effect of aging should the nurse assess for as a possible cause? Anosmia Xerostomia Hypochlorhydria Salivary gland tumor

Xerostomia Xerostomia (decreased saliva production), or dry mouth, affects many older adults and may be associated with difficulty swallowing (dysphagia). Anosmia is loss of sense of smell. Hypochlorhydria, a decrease in stomach acid, does not affect swallowing. Salivary gland tumors are not common.

An older adult patient reports difficulty swallowing. Which age-related change does the nurse teach the patient about? a. Xerostomia b. Esophageal cancer c. Decreased taste buds d. Thinner abdominal wall

Xerostomia Xerostomia, decreased volume of saliva, leads to dry oral mucosa and dysphagia. Esophageal cancer is not an age-related change. Decreased taste buds and a thinner abdominal wall do not contribute to difficulty swallowing.

Which information will the nurse include when teaching adults to decrease the risk for cancers of the tongue and buccal mucosa? a. Avoid use of cigarettes and smokeless tobacco. b. Use sunscreen when outside even on cloudy days. c. Complete antibiotic courses used to treat throat infections. d. Use antivirals to treat herpes simplex virus (HSV) infections.

a. Avoid use of cigarettes and smokeless tobacco. Tobacco use greatly increases the risk for oral cancer. Acute throat infections do not increase the risk for oral cancer, although chronic irritation of the oral mucosa does increase risk. Sun exposure does not increase the risk for cancers of the buccal mucosa. Human papillomavirus (HPV) infection is associated with an increased risk, but HSV infection is not a risk factor for oral cancer.

Which action should the nurse in the emergency department anticipate for a young adult patient who has had several episodes of bloody diarrhea? a. Obtain a stool specimen for culture. b. Administer antidiarrheal medication. c. Provide teaching about antibiotic therapy. d. Teach the adverse effects of acetaminophen (Tylenol).

a. Obtain a stool specimen for culture. Patients with bloody diarrhea should have a stool culture for Escherichia coli O157:H7. Antidiarrheal medications are usually avoided for possible infectious diarrhea to avoid prolonging the infection. Antibiotic therapy in the treatment of infectious diarrhea is controversial because it may precipitate kidney complications. Acetaminophen does not cause bloody diarrhea.

A 58-yr-old patient has just returned to the nursing unit after an esophagogastroduodenoscopy (EGD). Which action by unlicensed assistive personnel (UAP) requires that the registered nurse (RN) intervene? a. Offering the patient a pitcher of water b. Positioning the patient on the right side c. Checking the vital signs every 30 minutesd. d. Swabbing the patient's mouth with a wet cloth

a. Offering the patient a pitcher of water Immediately after EGD, the patient will have a decreased gag reflex and is at risk for aspiration. Assessment for return of the gag reflex should be done by the RN. The other actions by the UAP are appropriate.

A patient is being scheduled for endoscopic retrograde cholangiopancreatography (ERCP) as soon as possible. Which prescribed action should the nurse take first? a. Place the patient on NPO status. b. Administer sedative medications. c. Ensure the consent form is signed. d. Teach the patient about the procedure.

a. Place the patient on NPO status. The patient will need to be NPO for 8 hours before the ERCP is done, so the nurse's initial action should be to place the patient on NPO status. The other actions can be done after the patient is NPO.

A patient who underwent a gastroduodenostomy (Billroth I) 12 hours ago complains of increasing abdominal pain. The patient has no bowel sounds and 200 mL of bright red nasogastric (NG) drainage in the past hour. The highest priority action by the nurse is to a. contact the surgeon. b. irrigate the NG tube. c. monitor the NG drainage. d. administer the prescribed morphine.

a. contact the surgeon. Increased pain and 200 mL of bright red NG drainage 12 hours after surgery indicate possible postoperative hemorrhage, and immediate actions such as blood transfusion or return to surgery are needed (or both). Because the NG is draining, there is no indication that irrigation is needed. Continuing to monitor the NG drainage is not an adequate response. The patient may need morphine, but this is not the highest priority action.

The nurse will anticipate preparing an older patient who is vomiting "coffee-ground" emesis for a. endoscopy. b. angiography. c. barium studies. d. gastric analysis.

a. endoscopy. Endoscopy is the primary tool for visualization and diagnosis of upper gastrointestinal (GI) bleeding. Angiography is used only when endoscopy cannot be done because it is more invasive and has more possible complications. Barium studies are helpful in determining the presence of gastric lesions, but not whether the lesions are actively bleeding. Gastric analysis testing may help with determining the cause of gastric irritation, but it is not used for acute GI bleeding.

To palpate the liver during a head-to-toe physical assessment, the nurse a. places one hand on the patient's back and presses upward and inward with the other hand below the patient's right costal margin. b. places one hand on top of the other and uses the upper fingers to apply pressure and the bottom fingers to feel for the liver edge. c. presses slowly and firmly over the right costal margin with one hand and withdraws the fingers quickly after the liver edge is felt. d. places one hand under the patient's lower ribs and presses the left lower rib cage forward, palpating below the costal margin with the other hand.

a. places one hand on the patient's back and presses upward and inward with the other hand below the patient's right costal margin. The liver is normally not palpable below the costal margin. The nurse needs to push inward below the right costal margin while lifting the patient's back slightly with the left hand. The other methods will not allow palpation of the liver.

Which information about dietary management should the nurse include when teaching a patient with peptic ulcer disease (PUD)? a. "You will need to remain on a bland diet." b. "Avoid foods that cause pain after you eat them." c. "High-protein foods are least likely to cause you pain." d. "You should avoid eating any raw fruits and vegetables."

b. "Avoid foods that cause pain after you eat them." The best information is that each individual should choose foods that are not associated with postprandial discomfort. Raw fruits and vegetables may irritate the gastric mucosa, but chewing well seems to decrease this problem and some patients may tolerate these foods well. High-protein foods help neutralize acid, but they also stimulate hydrochloric (HCl) acid secretion and may increase discomfort for some patients. Bland diets may be recommended during an acute exacerbation of PUD, but there is little scientific evidence to support their use.

The nurse is assessing an alert and independent 78-yr-old patient for malnutrition risk. Which is the most appropriate initial question? a. "How do you get to the store to buy your food?" b. "Can you tell me the food that you ate yesterday?" c. "Do you have any difficulty in preparing or eating food?" d. "Are you taking any medications that alter your taste for food?"

b. "Can you tell me the food that you ate yesterday?" This question is the most open-ended and will provide the best overall information about the patient's daily intake and risk for poor nutrition. The other questions may be asked, depending on the patient's response to the first question.

Which information will the nurse include for a patient with newly diagnosed gastroesophageal reflux disease (GERD)? a. "Peppermint tea may reduce your symptoms." b. "Keep the head of your bed elevated on blocks." c. "You should avoid eating between meals to reduce acid secretion." d."Vigorous physical activities may increase the incidence of reflux."

b. "Keep the head of your bed elevated on blocks." Elevating the head of the bed will reduce the incidence of reflux while the patient is sleeping. Peppermint will decrease lower esophageal sphincter (LES) pressure and increase the chance for reflux. Small, frequent meals are recommended to avoid abdominal distention. There is no need to make changes in physical activities because of GERD.

Which information will the nurse include when teaching a patient with peptic ulcer disease about the effect of ranitidine (Zantac)? a. "Ranitidine absorbs the excess gastric acid." b. "Ranitidine decreases gastric acid secretion." c. "Ranitidine constricts the blood vessels near the ulcer." d. "Ranitidine covers the ulcer with a protective material."

b. "Ranitidine decreases gastric acid secretion." Ranitidine is a histamine-2 (H2) receptor blocker that decreases the secretion of gastric acid. The response beginning, "Ranitidine constricts the blood vessels" describes the effect of vasopressin. The response "Ranitidine absorbs the gastric acid" describes the effect of antacids. The response beginning "Ranitidine covers the ulcer" describes the action of sucralfate (Carafate).

Which area of the abdomen shown in the accompanying figure will the nurse palpate to assess for splenomegaly? a. 1 b. 2 c. 3 d. 4

b. 2 The spleen is usually not palpable, but when palpated, it is located in left upper quadrant of abdomen.

A young adult been admitted to the emergency department with nausea and vomiting. Which action could the RN delegate to unlicensed assistive personnel (UAP)? a. Auscultate the bowel sounds. c. b. Assist the patient with oral care. c. Assess for signs of dehydration. d. Ask the patient about the nausea.

b. Assist the patient with oral care. Oral care is included in UAP education and scope of practice. The other actions are all assessments that require more education and a higher scope of nursing practice.

A patient admitted with a peptic ulcer has a nasogastric (NG) tube in place. When the patient develops sudden, severe upper abdominal pain, diaphoresis, and a firm abdomen, which action should the nurse take? a. Irrigate the NG tube. b. Check the vital signs. c. Give the ordered antacid. d. Elevate the foot of the bed.

b. Check the vital signs. The patient's symptoms suggest acute perforation, and the nurse should assess for signs of hypovolemic shock. Irrigation of the NG tube, administration of antacids, or both would be contraindicated because any material in the stomach will increase the spillage into the peritoneal cavity. Elevating the foot of the bed may increase abdominal pressure and discomfort, as well as making it more difficult for the patient to breathe.

Which nursing action should be included in the postoperative plan of care for a patient after a laparoscopic esophagectomy? a. Reposition the NG tube if drainage stops. b. Elevate the head of the bed to at least 30 degrees. c. Start oral fluids when the patient has active bowel sounds. c. Notify the doctor for any bloody nasogastric (NG) drainage.

b. Elevate the head of the bed to at least 30 degrees. Elevation of the head of the bed decreases the risk for reflux and aspiration of gastric secretions. The NG tube should not be repositioned without consulting with the health care provider. Bloody NG drainage is expected for the first 8 to 12 hours. A swallowing study is needed before oral fluids are started.

A 49-yr-old man has been admitted with hypotension and dehydration after 3 days of nausea and vomiting. Which prescribed action will the nurse implement first? a. Insert a nasogastric (NG) tube. b. Infuse normal saline at 250 mL/hr. c. Administer IV ondansetron (Zofran). d. Provide oral care with moistened swabs.

b. Infuse normal saline at 250 mL/hr. Because the patient has severe dehydration, rehydration with IV fluids is the priority. The other orders should be accomplished after the IV fluids are initiated.

A 30-yr-old male patient with a body mass index (BMI) of 22 kg/m2 is being admitted to the hospital for elective knee surgery. Which assessment finding is important to report to the health care provider? a. Tympany on percussion of the abdomen b. Liver edge 3 cm below the costal margin c. Bowel sounds of 20/minute in each quadrant d. Aortic pulsations visible in the epigastric area

b. Liver edge 3 cm below the costal margin Normally the lower border of the liver is not palpable below the ribs, so this finding suggests hepatomegaly. The other findings are within normal range for the physical assessment.

The nurse is assessing a patient who had a total gastrectomy 8 hours ago. What information is most important to report to the health care provider? a. Hemoglobin (Hgb) 10.8 g/dL b. Temperature 102.1°F (38.9°C) c. Absent bowel sounds in all quadrants d. Scant nasogastric (NG) tube drainage

b. Temperature 102.1°F (38.9°C) An elevation in temperature may indicate leakage at the anastomosis, which may require return to surgery or keeping the patient NPO. The other findings are expected in the immediate postoperative period for patients who have this surgery and do not require any urgent action.

A patient returned from a laparoscopic Nissen fundoplication for hiatal hernia 4 hours ago. Which assessment finding is most important for the nurse to address immediately? a. The patient is experiencing intermittent waves of nausea. b. The patient has no breath sounds in the left anterior chest. c. The patient complains of 7/10 (0 to 10 scale) abdominal pain. d. The patient has hypoactive bowel sounds in all four quadrants.

b. The patient has no breath sounds in the left anterior chest. Decreased breath sounds on one side may indicate a pneumothorax, which requires rapid diagnosis and treatment. The nausea and abdominal pain should also be addressed, but they are not as high priority as the patient's respiratory status. The patient's decreased bowel sounds are expected after surgery and require ongoing monitoring but no other action.

The nurse is administering IV fluid boluses and nasogastric irrigation to a patient with acute gastrointestinal (GI) bleeding. Which assessment finding is most important for the nurse to communicate to the health care provider? a. The bowel sounds are hyperactive in all four quadrants. b. The patient's lungs have crackles audible to the midchest. c. The nasogastric (NG) suction is returning coffee-ground material. d. The patient's blood pressure (BP) has increased to 142/84 mm Hg.

b. The patient's lungs have crackles audible to the midchest. The patient's lung sounds indicate that pulmonary edema may be developing as a result of the rapid infusion of IV fluid and that the fluid infusion rate should be slowed. The return of coffee-ground material in an NG tube is expected for a patient with upper GI bleeding. The BP is slightly elevated but would not be an indication to contact the health care provider immediately. Hyperactive bowel sounds are common when a patient has GI bleeding.

Which information about an 80-yr-old male patient at the senior center is of most concern to the nurse? a. decreased appetite b. Unintended weight loss c. Difficulty chewing food d. Complaints of indigestion

b. Unintended weight loss Unintentional weight loss is not a normal finding and may indicate a problem such as cancer or depression. Poor appetite, difficulty in chewing, and complaints of indigestion are common in older patients. These will need to be addressed but are not of as much concern as the weight loss.

An older patient reports chronic constipation. To promote bowel evacuation, the nurse will suggest that the patient attempt defecation a. in the mid-afternoon. b. after eating breakfast. c. right after getting up in the morning. d. immediately before the first daily meal.

b. after eating breakfast. The gastrocolic reflex is most active after the first daily meal. Arising in the morning, the anticipation of eating, and physical exercise do not stimulate these reflexes.

A young adult patient is hospitalized with massive abdominal trauma from a motor vehicle crash. The patient asks the nurse about the purpose of receiving famotidine (Pepcid). The nurse will explain that the medication will a. decrease nausea and vomiting. b. inhibit development of stress ulcers. c. lower the risk for H. pylori infection. d. prevent aspiration of gastric contents.

b. inhibit development of stress ulcers. Famotidine is administered to prevent the development of physiologic stress ulcers, which are associated with a major physiologic insult such as massive trauma. Famotidine does not decrease nausea or vomiting, prevent aspiration, or prevent Helicobacter pylori infection.

An older patient with a bleeding duodenal ulcer has a nasogastric (NG) tube in place. The health care provider prescribes 30 mL of aluminum hydroxide/magnesium hydroxide (Maalox) to be instilled through the tube every hour. To evaluate the effectiveness of this treatment, the nurse a. monitors arterial blood gas values daily. b. periodically aspirates and tests gastric pH. c. checks each stool for the presence of occult blood. d. measures the volume of residual stomach contents.

b. periodically aspirates and tests gastric pH. The purpose for antacids is to increase gastric pH. Checking gastric pH is the most direct way of evaluating the effectiveness of the medication. Arterial blood gases may change slightly, but this does not directly reflect the effect of antacids on gastric pH. Because the patient has upper gastrointestinal bleeding, occult blood in the stools will appear even after the acute bleeding has stopped. The amount of residual stomach contents is not a reflection of resolution of bleeding or of gastric pH.

A 26-yr-old patient with a family history of stomach cancer asks the nurse about ways to decrease the risk for developing stomach cancer. The nurse will teach the patient to avoid a. emotionally stressful situations. b. smoked foods such as ham and bacon. c. foods that cause distention or bloating. d. chronic use of H2 blocking medications.

b. smoked foods such as ham and bacon. Smoked foods such as bacon, ham, and smoked sausage increase the risk for stomach cancer. Stressful situations, abdominal distention, and use of H2 blockers are not associated with an increased incidence of stomach cancer.

The nurse will plan to monitor a patient with an obstructed common bile duct for a. melena. b. steatorrhea. c. decreased serum cholesterol level. d. increased serum indirect bilirubin level.

b. steatorrhea. A common bile duct obstruction will reduce the absorption of fat in the small intestine, leading to fatty stools. Gastrointestinal bleeding is not caused by common bile duct obstruction. Serum cholesterol levels are increased with biliary obstruction. Direct bilirubin level is increased with biliary obstruction.

A 58-yr-old woman who was recently diagnosed with esophageal cancer tells the nurse, "I do not feel ready to die yet." Which response by the nurse is most appropriate? a."You may have quite a few years still left to live." b. "Thinking about dying will only make you feel worse." c. "Having this new diagnosis must be very hard for you." d. "It is important that you be realistic about your prognosis."

c. "Having this new diagnosis must be very hard for you." This response is open ended and will encourage the patient to further discuss feelings of anxiety or sadness about the diagnosis. Patients with esophageal cancer have a low survival rate, so the response "You may have quite a few years still left to live" is misleading. The response beginning, "Thinking about dying" indicates that the nurse is not open to discussing the patient's fears of dying. The response beginning, "It is important that you be realistic" discourages the patient from feeling hopeful, which is important to patients with any life-threatening diagnosis.

A patient who has gastroesophageal reflux disease (GERD) is experiencing increasing discomfort. Which patient statement to the nurse indicates that additional teaching about GERD is needed? a. "I take antacids between meals and at bedtime each night." b. "I sleep with the head of the bed elevated on 4-inch blocks." c. "I eat small meals during the day and have a bedtime snack." d. "I quit smoking several years ago, but I still chew a lot of gum."

c. "I eat small meals during the day and have a bedtime snack." GERD is exacerbated by eating late at night, and the nurse should plan to teach the patient to avoid eating at bedtime. The other patient actions are appropriate to control symptoms of GERD.

Which statement to the nurse from a patient with jaundice indicates a need for teaching? a. "I used cough syrup several times a day last week." b. "I take a baby aspirin every day to prevent strokes." c. "I use acetaminophen (Tylenol) every 4 hours for back pain." d. "I need to take an antacid for indigestion several times a week"

c. "I use acetaminophen (Tylenol) every 4 hours for back pain." Chronic use of high doses of acetaminophen can be hepatotoxic and may have caused the patient's jaundice. The other patient statements require further assessment by the nurse but do not indicate a need for patient education.

Which patient should the nurse assess first after receiving change-of-shift report? a. A patient with nausea who has a dose of metoclopramide (Reglan) due b. A patient who is crying after receiving a diagnosis of esophageal cancer c. A patient with esophageal varices who has a blood pressure of 92/58 mm Hg d. A patient admitted yesterday with gastrointestinal (GI) bleeding who has melena

c. A patient with esophageal varices who has a blood pressure The patient's history and blood pressure indicate possible hemodynamic instability caused by GI bleeding. The data about the other patients do not indicate acutely life-threatening complications.

Which finding by the nurse during abdominal auscultation indicates a need for a focused abdominal assessment? a. Loud gurgles b. High-pitched gurgles c. Absent bowel sounds d. Frequent clicking sounds

c. Absent bowel sounds Absent bowel sounds are abnormal and require further assessment by the nurse. The other sounds may be heard normally.

Which prescribed action will the nurse implement first for a patient who has vomited 1100 mL of blood? a. Give an IV H2 receptor antagonist. b. Draw blood for typing and crossmatching. c. Administer 1 L of lactated Ringer's solution. d. Insert a nasogastric (NG) tube and connect to suction.

c. Administer 1 L of lactated Ringer's solution. Because the patient has vomited a large amount of blood, correction of hypovolemia and prevention of hypovolemic shock are the priorities. The other actions also are important to implement quickly, but are not the highest priorities.

An adult with Escherichia coli O157:H7 food poisoning is admitted to the hospital with bloody diarrhea and dehydration. Which prescribed action will the nurse question? a. Infuse lactated Ringer's solution at 250 mL/hr. b. Monitor blood urea nitrogen and creatinine daily. c. Administer loperamide (Imodium) after each stool. d. Provide a clear liquid diet and progress diet as tolerated.

c. Administer loperamide (Imodium) after each stool. Use of antidiarrheal agents is avoided with this type of food poisoning. The other orders are appropriate.

A 53-yr-old male patient with deep partial-thickness burns from a chemical spill in the workplace experiences severe pain followed by nausea during dressing changes. Which action will be most useful in decreasing the patient's nausea? a. Keep the patient NPO for 2 hours before dressing changes. b.Give the ordered prochlorperazine before dressing changes. c.Administer the prescribed morphine sulfate before dressing changes. d.Avoid performing dressing changes close to the patient's mealtimes.

c. Administer the prescribed morphine sulfate before dressing changes Because the patient's nausea is associated with severe pain, it is likely that it is precipitated by stress and pain. The best treatment will be to provide adequate pain medication before dressing changes. The nurse should avoid doing painful procedures close to mealtimes, but nausea or vomiting that occur at other times also should be addressed. Keeping the patient NPO does not address the reason for the nausea and vomiting and will have an adverse effect on the patient's nutrition. Administration of antiemetics is not the best choice for a patient with nausea caused by pain. However, an antiemetic may be added later if the nausea persists despite pain management.

Which patient choice for a snack 3 hours before bedtime indicates that the nurse's teaching about gastroesophageal reflux disease (GERD) has been effective? a. Chocolate pudding b. Glass of low-fat milk c. Cherry gelatin with fruit d. Peanut butter and jelly sandwich

c. Cherry gelatin with fruit Gelatin and fruit are low fat and will not decrease lower esophageal sphincter (LES) pressure. Foods such as chocolate are avoided because they lower LES pressure. Milk products increase gastric acid secretion. High-fat foods such as peanut butter decrease both gastric emptying and LES pressure.

After the nurse has completed teaching a patient with newly diagnosed eosinophilic esophagitis about the management of the disease, which patient action indicates that the teaching has been effective? a. Patient orders nonfat milk for each meal. b. Patient uses the prescribed corticosteroid inhaler. c. Patient schedules an appointment for allergy testing. d. Patient takes ibuprofen (Advil) to control throat pain.

c. Patient schedules an appointment for allergy testing. Eosinophilic esophagitis is frequently associated with environmental allergens, so allergy testing is used to determine possible triggers. Corticosteroid therapy may be prescribed, but the medication will be swallowed, not inhaled. Milk is a frequent trigger for attacks. NSAIDs are not used for eosinophilic esophagitis.

Which finding in the mouth of a patient who uses smokeless tobacco is suggestive of oral cancer? a. Bleeding during tooth brushing b. Painful blisters at the lip border c. Red, velvety patches on the buccal mucosa d. White, curdlike plaques on the posterior tongue

c. Red, velvety patches on the buccal mucosa A red, velvety patch suggests erythroplasia, which has a high incidence (>50%) of progression to squamous cell carcinoma. The other lesions are suggestive of acute processes (e.g., gingivitis, oral candidiasis, herpes simplex).

A patient has just arrived in the recovery area after an upper endoscopy. Which information collected by the nurse is most important to communicate to the health care provider? a. The patient is very drowsy. b. The patient reports a sore throat. c. The oral temperature is 101.4°F. d. The apical pulse is 100 beats/minute.

c. The oral temperature is 101.4°F. A temperature elevation may indicate that an acute perforation has occurred. The other assessment data are normal immediately after the procedure.

A 58-yr-old patient has just been admitted to the emergency department with nausea and vomiting. Which information requires the most rapid intervention by the nurse? a. The patient has been vomiting for 4 days. b. The patient takes antacids 8 to 10 times a day. c. The patient is lethargic and difficult to arouse. d. The patient has had a small intestinal resection.

c. The patient is lethargic and difficult to arouse. A lethargic patient is at risk for aspiration, and the nurse will need to position the patient to decrease aspiration risk. The other information is also important to collect, but it does not require as quick action as the risk for aspiration.

A 38-year old woman receiving chemotherapy for breast cancer develops a Candida albicans oral infection. The nurse will anticipate the need for a. hydrogen peroxide rinses. b. the use of antiviral agents. c. administration of nystatin tablets. d. referral to a dentist for professional tooth cleaning.

c. administration of nystatin tablets. Candida albicans infections are treated with an antifungal such as nystatin. Peroxide rinses would be painful. Oral saltwater rinses may be used but will not cure the infection. Antiviral agents are used for viral infections such as herpes simplex. Referral to a dentist is indicated for gingivitis but not for Candida infection.

A 68-yr-old male patient with a stroke is unconscious and unresponsive to stimuli. After learning that the patient has a history of gastroesophageal reflux disease (GERD), the nurse will plan to do frequent assessments of the patient's a. apical pulse. b. bowel sounds. c. breath sounds d. abdominal girth.

c. breath soundsBecause GERD may cause aspiration, the unconscious patient is at risk for developing aspiration pneumonia. Bowel sounds, abdominal girth, and apical pulse will not be affected by the patient's stroke or GERD and do not require more frequent monitoring than the routine.

At his first postoperative checkup appointment after a gastrojejunostomy (Billroth II), a patient reports that dizziness, weakness, and palpitations occur about 20 minutes after each meal. The nurse will teach the patient to a. increase the amount of fluid with meals. b. eat foods that are higher in carbohydrates. c. lie down for about 30 minutes after eating. d. drink sugared fluids or eat candy after meals.

c. lie down for about 30 minutes after eating. The patient is experiencing symptoms of dumping syndrome, which may be reduced by lying down after eating. Increasing fluid intake and choosing high carbohydrate foods will increase the risk for dumping syndrome. Having a sweet drink or hard candy will correct the hypoglycemia that is associated with dumping syndrome but will not prevent dumping syndrome.

After assisting with a needle biopsy of the liver at a patient's bedside, the nurse should a. put pressure on the biopsy site using a sandbag. b. elevate the head of the bed to facilitate breathing. c. place the patient on the right side with the bed flat. d. check the patient's post-biopsy coagulation studies.

c. place the patient on the right side with the bed flat. After a biopsy, the patient lies on the right side with the bed flat to splint the biopsy site. Coagulation studies are checked before the biopsy. A sandbag does not exert adequate pressure to splint the site

While interviewing a young adult patient, the nurse learns that the patient has a family history of familial adenomatous polyposis (FAP). The nurse will plan to assess the patient's knowledge about a. preventing noninfectious hepatitis. b. treating inflammatory bowel disease. c. risk for developing colorectal cancer. d. using antacids and proton pump inhibitors.

c. risk for developing colorectal cancer. FAP is a genetic condition that greatly increases the risk for colorectal cancer. Noninfectious hepatitis, use of medications that treat increased gastric pH, and inflammatory bowel disease are not related to FAP.

When caring for a patient with a history of a total gastrectomy, the nurse will monitor for a. constipation. b. dehydration. c. elevated total serum cholesterol. d. cobalamin (vitamin B12) deficiency.

cobalamin (vitamin B12) deficiency. The patient with a total gastrectomy does not secrete intrinsic factor, which is needed for cobalamin (vitamin B12) absorption. Because the stomach absorbs only small amounts of water and nutrients, the patient is not at higher risk for dehydration, elevated cholesterol, or constipation.

An 80-yr-old patient who is hospitalized with peptic ulcer disease develops new-onset auditory hallucinations. Which prescribed medication will the nurse discuss with the health care provider before administration? a. Sucralfate (Carafate) b. Aluminum hydroxide c. Omeprazole (Prilosec) d. Metoclopramide (Reglan)

d. Metoclopramide (Reglan) Metoclopramide can cause central nervous system side effects ranging from anxiety to hallucinations. Hallucinations are not a side effect of proton pump inhibitors, mucosal protectants, or antacids.

A 73-yr-old patient is diagnosed with stomach cancer after an unintended 20-lb weight loss. Which nursing action will be included in the plan of care? Refer the patient for hospice services. a. Infuse IV fluids through a central line. c. Teach the patient about antiemetic therapy. d. Offer supplemental feedings between meals.

d. Offer supplemental feedings between meals. The patient data indicate a poor nutritional state and improvement in nutrition will be helpful in improving the response to therapies such as surgery, chemotherapy, or radiation. Nausea and vomiting are not common clinical manifestations of stomach cancer. There is no indication that the patient requires hospice or IV fluid infusions.

Which assessment should the nurse perform first for a patient who just vomited bright red blood? a. Measuring the quantity of emesis b. Palpating the abdomen for distention c. Auscultating the chest for breath sounds d. Taking the blood pressure (BP) and pulse

d. Taking the blood pressure (BP) and pulse The nurse is concerned about blood loss and possible hypovolemic shock in a patient with acute gastrointestinal bleeding. BP and pulse are the best indicators of these complications. The other information is important to obtain, but BP and pulse rate are the best indicators for assessing intravascular volume.

A 42-yr-old patient is admitted to the outpatient testing area for an ultrasound of the gallbladder. Which information obtained by the nurse indicates that the ultrasound may need to be rescheduled? a. The patient took a laxative the previous evening. b. The patient had a high-fat meal the previous evening. c. The patient has a permanent gastrostomy tube in place. d. The patient ate a low-fat bagel 4 hours ago for breakfast.

d. The patient ate a low-fat bagel 4 hours ago for breakfast. Food intake can cause the gallbladder to contract and result in a suboptimal study. The patient should be NPO for 8 to 12 hours before the test. A high-fat meal the previous evening, laxative use, or a gastrostomy tube will not affect the results of the study.

The nurse receives the following information about a 51-yr-old female patient who is scheduled for a colonoscopy. Which information should be communicated to the health care provider before sending the patient for the procedure? a. The patient has a permanent pacemaker to prevent bradycardia. b. The patient is worried about discomfort during the examination. c. The patient has had an allergic reaction to both shellfish and iodine in the past. d. The patient declined to drink the prescribed polyethylene glycol (GoLYTELY).

d. The patient declined to drink the prescribed polyethylene glycol (GoLYTELY). If the patient has had inadequate bowel preparation, the colon cannot be visualized and the procedure should be rescheduled. Because contrast solution is not used during colonoscopy, the iodine allergy is not pertinent. A pacemaker is a contraindication to magnetic resonance imaging but not to colonoscopy. The nurse should instruct the patient about the sedation used during the examination to decrease the patient's anxiety about discomfort.

The health care provider prescribes antacids and sucralfate (Carafate) for treatment of a patient's peptic ulcer. The nurse will teach the patient to take a. sucralfate at bedtime and antacids before each meal. b. sucralfate and antacids together 30 minutes before meals. c. antacids 30 minutes before each dose of sucralfate is taken. d. antacids after meals and sucralfate 30 minutes before meals.

d. antacids after meals and sucralfate 30 minutes before meals. Sucralfate is most effective when the pH is low and should not be given with or soon after antacids. Antacids are most effective when taken after eating. Administration of sucralfate 30 minutes before eating and antacids just after eating will ensure that both drugs can be most effective. The other regimens will decrease the effectiveness of the medications.

A patient who requires daily use of a nonsteroidal antiinflammatory drug (NSAID) for the management of severe rheumatoid arthritis has recently developed melena. The nurse will anticipate teaching the patient about a. substitution of acetaminophen (Tylenol) for the NSAID. b. use of enteric-coated NSAIDs to reduce gastric irritation. c. reasons for using corticosteroids to treat the rheumatoid arthritis. d. misoprostol (Cytotec) to protect the gastrointestinal (GI) mucosa.

d. misoprostol (Cytotec) to protect the gastrointestinal (GI) mucosa. Misoprostol, a prostaglandin analog, reduces acid secretion and the incidence of upper GI bleeding associated with NSAID use. Enteric coating of NSAIDs does not reduce the risk for GI bleeding. Corticosteroids increase the risk for ulcer development and will not be substituted for NSAIDs for this patient. Acetaminophen will not be effective in treating rheumatoid arthritis.

A patient vomiting blood-streaked fluid is admitted to the hospital with acute gastritis. To determine possible risk factors for gastritis, the nurse will ask the patient about a. the amount of saturated fat in the diet. b. a family history of gastric or colon cancer. c. a history of a large recent weight gain or loss. d. use of nonsteroidal antiinflammatory drugs (NSAIDs).

d. use of nonsteroidal antiinflammatory drugs (NSAIDs). Use of an NSAID is associated with damage to the gastric mucosa, which can result in acute gastritis. Family history, recent weight gain or loss, and fatty foods are not risk factors for acute gastritis.


Conjuntos de estudio relacionados

Federal Tax Considerations for Life Insurance and Annuities

View Set

Elbow, Radioulnar, Wrist, and Hand Joints Practice

View Set